SlideShare una empresa de Scribd logo
1 de 174
Descargar para leer sin conexión
OSCE–kkcth

             Question No : 1




List 4 findings on the x- ray

a)   What is the radiological diagnosis?
b)     List 4 causes for the finding in the left

     lung

c)   List 4 causes for the finding in the left lung




     Answer for Question No : 1

a)   Mediastinal shift (L)
     Pneumothorax (L)
     Atlectasis (L) lung
     Bronchopnemonic changes (R)

b)   Pneumothorax
     Bronchopneumonic changes (R)
      (Non – homogenous opacities

c) 1) Pneumonia
   2) Asthma
   3) Foreign body in lung
   4) Trauma
5) Cystic fibrosis
  6) Tracheotomy
  7) Subclavian line present
  8) Thorococentesis
  9) Transbronchial biopsy
  10) Lymphomas & other malignancies
  11) Ehlers Danlos syndrome
  12) Marfan syndrome
  13) Gangrene
  14) Lung abscess
              Question No : 2

LINEZOLID
    1)   What group of drug is this?

    2)   Mode of action

    3)   Name 2 organisms for which this is a

         specific drug?

    4)   Dosage
5)    Mention important hematological   side

           effect?




          Answer for Question No : 2

1)   Oxazolidinone

2)   Translation initiation by blocking, formation

     of protein synthesis initiation complex by

     binding to 50 S ribosomal RNA.
3)   MRSA vancomycin resistant Enterococci,

      coagulase negative staphylococci, penicillin

      resistant pneumococci

 4)   10mg / kg / dose Q8 –12 hrs

 5)   Thrombocytopenia




                Question No : 3

QUINUPRISTIN / DALFOPRISTIN

 1)   What group of drug is this?

 2)   What is the mode of action?

 3)   Against which of the following organisms

      this is in effective?
Mycoplasma, chlamydia, staph. aureus

     enterococcus faecalis.




     Answer for Question No: 3

1)   Streptogramins    Strepto gramins

2)   Synergistic action on bacterial ribosomal

     subunit to protein synthesis

3)   Enterococcus faecalis.
Question No : 4

pH      - 7.38

Pco2 - 38

BE      -3

Hco3 - 21

Po2     - 98

1)    Interpret the ABG?

2)    List the indicators of compensation in ABG

      in following conditions.
a)   Metabolic Acidosis

         b)   Metabolic Alkalosis

         c)   Respiratory Acidosis – Acute/ Chronic

         d) Respiratory Alkalosis – Acute/ Chronic




         Answer for Question No : 4
1)       Normal ABG

        Met. Acidosis PCo2 = 15 x (HCo3) +8+/-2

        Metabolic Alkalosis - PaCo2 increases by

         7mm of Hg for each 10mq increases in the

         (HCo3 -)

        Respiratory    Acidosis    Acute   –   (HCo3)

         increases by 1 for each 10mm increased in

         Pco2.

         Chronic – (Hco3) increases by 3.5 for each
10mm increase in PCo2.

        Respiratory Alkalosis Acute – (HCo3) fall by

         2 for each 10mm Hg decrease in PCo2.

         Chronic – (HCo3) decreased by 4 for each

         10mm of decrease in PCo2.




                               Question No : 5
1) You are asked to counsel a mother who’s 9 month of

     infant has AWD regarding ORT

Check list:

         i)          Introduces himself

         ii)         Explains that the main treatment is ORT and explains

                     the need for rehydration.

         iii)        Explains correctly the preparation of ORT

                     whole packet in 1 liter of water.

         iv)         Advises feeding by spoon discourages bottle feeding.

2)               Mother asks what to do if the baby vomits

         v) Stop ORT for 5 – 10minutes and restart feed, give slowly

                 spoonful every 2 – 3 minutes

               vi)    Advise giving small aliquots of 5 – 10ml each time.
vii)     Explains the danger signs of dehydration and

               explains when she should seek medical attention

               Does not become better in 3 days or develops danger

               Signs (Seizure / unconscious / rapid breathing etc).

      viii) Encourage continuance of breast feeds / normal feeds /

                   home available feeds.

       ix)     Checks, whether the mother has understood or not.

       x)      Ask the mother whether there are any doubts.


                     Answer for Question No : 5
1) You are asked to counsel a mother who’s 9 month of

     infant has AWD regarding ORT

Check list:

       a. Introduces himself

       b. Explains that the main treatment is ORT and explains the

             need for rehydration.

       c. Explains correctly the preparation of ORT

                   whole packet in 1 liter of water.

       d. Advises feeding by spoon discourages bottle feeding.

2)             Mother asks what to do if the baby vomits

       v) Stop ORT for 5 – 10minutes and restart feed, give slowly

               spoonful every 2 – 3 minutes

             vi)     Advise giving small aliquots of 5 – 10ml each time.
vii)   Explains the danger signs of dehydration and

              explains when she should seek medical attention

              Does not become better in 3 days or develops danger

              Signs (Seizure / unconscious / rapid breathing etc).

       viii) Encourage continuance of breast feeds / normal feeds /

              home available feeds.

       ix)    Checks, whether the mother has understood or not.

        x)    Ask the mother whether there are any doubts.




                     Question No : 6

6 yrs old boy is brought for bed-wetting. His

frequency in day time is normal. He is dry in the

day. He is never been dry in the night.

  1)   Define this problem? What type is it?
2)   Is   it       complicated       or    uncomplicated?

     Enumerate          4     differences         between

     complicated and uncomplicated.

3)   Name      3     drugs    and     dosage      for   the

     pharmacological therapy of this condition.

     Name 3 non – pharmacological measures for

     the management of this condition.


     Answer for Question No : 6
1)   Nocturnal Enuresis. Primary Nocturnal Enuresis

2)   Uncomplicated

                    Uncomplicated         Complicated

Onset                   Primary            Secondary

Daytime symptoms        absent                +

Stream                      Normal          Abnormal
Physical                 Normal        Abnormal

 Urine analysis           Normal        Abnormal

 3)            Drug “ODI!!!”             Dose

              DDAVP            10 – 40 mcg/day

                               Nasal spray

             Oxybutinin        10 - 20 mg/day PO

            Imipramine         0.9 – 1.5 mg/kg/day

Non-pharmacological

Behavioral modification, Bladder exercises, alarm device.


                  Question No : 7


 List 3 abnormalities in this ECG

 1)   What is the ECG Diagnosis?

 2)   List 4 causes for the same
3)   Drug of choice

4)   Mention 1 complication




     Answer for Question No : 7

1)   Fibrillary waves

     Absence of P waves

     Irregular Ventricular response / rhythm

2)   Atrial Fibrillation

3)   Rheumatic Valvular disease
Thyrotoxicosis

     Following cardiac surgery

     Pulmonary embolism

     Pericarditis

     WPW syndrome

     Mitral regurgitation

4)   Digoxin

5)   Stroke / Thromboembolism




               Question No: 8
1) Identify the organism?

2) Name the method and steps used for the

   preparation for the smear?
Answer for the Question No: 8

Acid-fast bacilli

Ziehl neelsen technique     CSEWM

Heat and dry. Fix the smear.

Add strong carbol fushcin

Heat approximately for 5mins. Do not boil.

Decolorise the smear with 20% sulphuric acid

Decolorise with ethanol

Wash with water

Counter stain with methylene blue
Question No : 9




1)   Describe the pedigree

2)   What is the mode of inheritance?

3)   Give 4 examples.
Answer for Question No : 9
1)       3 generation pedigree chart showing

      All daughters of the affected males have

         the disease

      Sons of the affected males are normal

      Affected females affect ½ of the males

         and ½ of the daughters

2)       X- linked dominant inheritance

3)       Hypophosphatemic rickets ( Vit.D resistant)

        Incontinentia pigmenti        XD RHIO

        Oro facial digital syndrome

        Rett syndrome
Question No : 10

The following food substances, which contain Vit.A,

need to be arranged based on Vitamin A content

from high to low.

   Papaya,

   Guava

   Amaranth

   Drumstick leaves

   Egg

   Human milk

   Carrot
Answer for Question No : 10

               VIT A : CADEPM

   Carrot           1167

 Amaranth           515

 Drumstick leaves   300

 Egg                140

 Papaya             118

 Human Milk         38

 Guava              0
Question No : 11

1) Vitamin A prophylaxis programme in India -

 Mention the dosage and schedule.

2) Daily requirement of Vitamin A.

3) Name two manifestation of hypervitaminosis A?
Answer for Question No : 11
1) 5 doses 9 months – 3 yrs
Oral retinol palmitate
 1lakh 9 months (along with measles)
 2lakh 1½ yrs
 2lakh 2 yrs
 2lakh 2 ½ yrs
 2lakh 3 yrs
2) 400mg to 600mg of requirement
          Retinal / RE B –carotene I.U
0-1yr       350mcg        1200mcg       1166.67
1-6yrs      400mcg        1600mcg      1333.33
>7yrs       600mcg        8400mcg       2000
   1mcg = 3.3 IU
   1 IU of vitamin A = 0.3mcg at retinal
3) Nausea, vomiting,
      anorexia, sleep distress, irritability
      Skin desquamation
      Hepatomegaly
      Pseudo tumor cerebri (diplopia/ papilledema
      /cranial N.Palsy)
     Alopecia, seborrhea,
cutaneous leisions craniotabes
     Tender bony swellings
     Fissures at corners and mouth




            Question No : 12

Match the following:

  1) BCG               - Toxoid & killed bacteria

  2) OPV               -     Live attenuated bacteria

  3) DPT               - Bacterial sub unit

  4) Hib               - Viral Antigen

  5) Hep B Vaccine     –     Live attenuated viral

  6) Typhoid V I           - Killed virus

  7) Hep A Vaccine         – Capsular polysaccharide

  8) Acellular pertusis – Capsular polysaccharide
Question No : 13


Answer the following:

  What is the Diluent for BCG?

  What is the Diluent for MMR?

  How long can reconstituted BCG be used?

  How long can reconstituted MMR be used?

  Name 5 vaccines which should not be frozen

  What does IAP recommend at 5 yrs -

    (DPT/DT)?
Answer for Question No: 12

1) BCG                -       Live attenuated bacteria

2) OPV                    -   Live attenuated viral

3) DPT                -       Toxoid & killed bacteria

4) Hib                -       Capsular poly saccharide

5) Hep B Vaccine      -       Viral Antigen

6) Typhoid V I        -       Capsular poly saccharide

7) Hep A Vaccine      -       Killed virus

8) Acellular pertusis -       Bacterial sub unit
Answer for Question No : 13
 Diluent for BCG is Sterile NS

 Diluent for MMR is Distilled water

 Reconstituted BCG can be used for 3 hrs

 Reconstituted MMR can be used for 1 hr.

 DPT, Hepatitis A & B, Varicella, Hib, TT

 IAP recommends DPT at 5 yrs.
What are indications for Acellular pertusis –

vaccine?

Mention 4 indications for pneumococcal vaccine?
Answer for Question No : 13

Indications are:

   Persistent / inconsolable Cry 3 or more hrs in 48 hrs.

 Temperature > 40º within 48 hrs
 Collapse / shock with (HHE within 48 hrs)
 Convulsions with or without fever within 72 hrs of
    immunization.

 Encephalopathy within 7 days, behavioral problems.


3) Indications:

 Prior to splenectomy,
 HIV

 CSF Rhinorrhea

 Sickle cell

 Asplenia
 CRF

     Chronic lung / heart disease




      Question No : 14

A 6 year old girl has been referred for evaluation

of anemia. Answer the questions after seeing the

peripheral smear?




a) What is your diagnosis?
b) What would be the confirmatory test to

  clinch your diagnosis ?

c) What is the definitive treatment of this

  condition ?




      Answer for Question No : 14


 1)   Hereditary spherocytosis

 2)   Incubated osmotic fragility

 3)   Splenectomy
Question No : 15

You are asked to resuscitate a newborn with the

provided equipments.

Please ask questions regarding status of infant –

wherever necessary.

  1)   Check the following equipments before

       proceeding further

        Bag mask valve

        Laryngoscope

  2)   Get information about the infant from the

       observer before proceeding to resuscitate

       and at each step whenever necessary




           Answer for Question No : 15
1) Check list for observer.
       Bag mask valve…does he
                         - attach reservoir?
                            - check pop off valve?
                                 - check for leak?
   Laryngoscope – Checks bulb & handle
   Does candidate ask the following 5 questions?
   Meconium staining of liquor or not?
   Term or preterm?
   Crying well – breathing well or not?
   Pink or blue colour?
   Good muscle tone?

The Observer Should Say Baby Is Not Breathing
Does he clear airway/provide warmth/ position dry infant?
1..5 marks
   and then ask status of baby

Observer Says: Baby Still Not Breathing well
  Does he give PPV for 30 seconds?
         Correct position
         EC clamp technique
         Chest expansion
  and then ask status of baby
  Observer Says Hr- 50/Min, Blue

  Does he start chest compressions?
  Correct technique?
Question No : 16




1)   What is the diagnosis?

2)   This infant is 8 months old, what is the most

     likely type?



3)   What is the earliest sign of this disorder?

4)   What is the first radiological change that

     occurs in response to specific therapy?

5)   How could this have been prevented?



6)   What are the non – specific urinary findings
in this disorder? (at least 2)




     Answer for Question No: 16

1)   Rickets

2)   Vitamin D deficiency

3)   Craniotabes

4)   Appearance       of    provisional   zone   of

     calcification

5)   Supplement of 400IU of vitamin D

6)   Generalized aminoaciduria

      Glycosuria

      Phosphaturia

      Elevated urinary citrate
 Impaired renal acidification.




               Question No : 17

You have performed ICD on a child with

empyema. How will you dispose the used items

given below?

   Scalpel blade, hypodermic needles, trochar,

    used ampoules

   Cotton,     gauze,   linens,   suture   material,

    surgical mask, gloves

   Pus, 3 way connector

   Syringe, plastic covers of gloves and ICD

    bag cover
Answer for Question No : 17



     Blue / white transparent puncture proof

    container

 Yellow bag

 Red bag

 Black plastic bag.
Question No : 18




This child has fever with URI

  1)   What is the diagnosis?

  2)   What is the causative organism?

  3)   Name one serious hematological complication

       in this disease.

  4)   Name one orthopedic complication.

  5)   Name one cause for intrauterine fetal

       demise.
6)   What treatment is recommended for severe

     hematological complications?




     Answer for Question No : 18




1)   Erythema Infection or fifth disease

2)   Parvovirus B 19

3)   Transient aplastic crisis

4)   Arthropathy

5)   Non – immune fetal hydrops

6)   IvIg
Question No : 19




Calculate the mean, median, mode and mean

deviation of the diastolic pressures given

below.

   83,75,81,79,71,95,75,77,84
Answer for Question No : 19




Mean = 80, Median = 79, Mode = 75 and

Mean deviation = 5.1

The average of the deviations from

arithmetic mean
   MD = ∑ (x -xˉ)/n   ∑      = summation
                      x      = item values
                      xˉ     = Mean
               x - xˉ = deviation from mean
                      n      = No. of items
Question No :20

Take relevant history from this parent

whose child is suspected to have urinary

tract infection for the first time.
Answer for Question No : 20

Introduces himself

History of fever

History of constipation

History of urgency

History of malodorous urine

History of suprapubic pain

History of loin pain

Details of coevute toilet training

Wiping from back to front

History of incontinence

History of threadworm infection

Family history of renal disease / stones
Family history of UTI / VUR

Note of thanks




              Question No : 21




You are asked to perform rapid sequence

intubation.

Write the steps sequentially.

Mention the names of drugs wherever

necessary.
Answer for Question No : 21

      Brief history and assessment

      Assemble equipment, medications, etc.

      Preoxygenate patient

      Premedicate with lidocaine atropine

      Sedation and analgesia induced

      Pretreat with nondepolarizing paralytic agent

      Administer muscle relaxants

      Sellick maneuver

      Endotracheal intubation

      Secure tube, verify position with roentgenogram

      Begin mechanical ventilation

Step – 5 : Sedatives:

Thiopental

Diazepam

Ketamine

Analgesics:

Fentanyl

Morphine

Succinylcholine

Vecuronium or Pancuronium or rocuronium
Question No : 22

  A 2 year child presents with the following

        5 episodes of abscesses in 6 months

        Photosensitivity

        Light skin and silvery hair

Peripheral smear shows large inclusions in all

nucleated blood cells.

  1)     What is the diagnosis?

  2)     What is the cause for the lighten hair?

  3)     What is the mode of inheritance?

  4)     Name one life threatening hematological

         complications?

  5)     What is the neurological manifestation?
6)   Which drug is indicated?

     Answer for Question No : 22
1)   Chediak – Higashi syndrome

2)   Melanosomes or melanocytes are oversized.

     Failure to properly disperse the giant

     melanosomes to keratinocytes and hair

     follicles.

3)   Autosomal recessive. Mutated gene for CHS

     Chromosome 1q2-q44.

4)   Accelerated phase of a lymphoma like

     syndrome characterized by pancyopenia.

5)   Peripheral neuropathy and ataxia

     Motor        Sensory

6)   High – dose ascorbic acid

200mg / 24hrs for infants

2,000mg/24hrs for adults. (2g !)
Question No : 23




Perform Hand Washing
Answer for Question No : 23

a)   Remove ornaments / watch etc. hand

     sleeves above elbows.

b)   Perform six steps of hand washing

1)   Palm to Palm

2)   (i) Right palm over left dorsum

     (ii) Left palm over right dorsum

3)   Fingers interlace palm to palm

4)   Back of fingers to opposing palms

5)   (i) Rotational rubbing of right thumb

     (ii) Rotational rubbing of left thumb

6)   (i) Rotational rubbing of left palm

     (ii) Rotational rubbing of right palm

c)   Perform in 2 minutes

d)   Air dry / dry with sterile towel / paper

e)   Discard towel in black cover.
Question No : 24
1) A Lumbar puncture is performed and

   the CSF is xanthochromatic. What are

   the four possible causes?

2) CSF protein levels are 400mg/dl. What

   are the three possible causes for the

   same?

3) CSF Glucose in 200mg/dl and blood

   glucose is 112mg/dl. List five causes for

   the same.

4) CSF is also cloudy, what does it imply?
Answer for Question No : 24
1)   a) Hyperbilirubinemia

     b) Subarachnoid hemorrhage

     c) Markedly elevated CSF protein

     d)   Carotenemia

 2) a) TB Meningitis

     b) GBS

     c) Tumors of spinal cord / brain

     d) Degenerative disorders

     e) Vasculitis

     f) Multiple sclerosis

3)   a) Bacterial meningitis

     b) TBM

     c) Fungal meningitis

     d) Aseptic meningitis

     f) Neoplasms of meninges

4)   Elevated WBC or RBC count
Question No : 25
An adolescent presents with history of

ingestion   insecticides   and     has   clinical

features of organophosphorus poisoning.

 1) Mention 2 methods of decontamination

     needed.

 2) What is the mode of action of 2

     antidotes used?

 3) What 2 laboratory parameters are used

     to     confirm    the       diagnosis    of

     organophosphorus poisoning?
Answer for Question No : 25
1)   Activated           charcoal       for         gastric

     decontamination. Skin decontamination by

     removal       of      clothes      stained       with

     organophosphorus.

2)   Atropine – blocks acetylcholine receptor.

     Reverses the muscarinic and CNS effects.

     Pralidoxime     (PAM)     –     breaks   the     bond

     between       the    organophosphate       and    the

     enzyme, liberating the enzyme and degrading

     the organophosphate.

3)   Red    cell     cholinesterase       and       pseudo

     cholinesterase levels.
Question No : 26

Answer the following questions after seeing

the X-ray.

 1) What are the findings?

 2) Name 5 aerobic organisms, which can

     cause this?
Answer for Question No : 26

1) Air and fluid filled cyst.

  Lung abscess, pneumatocele

2) Streptococcus

   Staphylococcus aureus

   Escherichia coli

   Klebsiella

   Pseudomonas
Question No : 27

Dobutamine

 1) What is the mode of action?

 2) 7kg child requires Dobutamine infusion.

    How do you prepare the infusion? What

    is the dose?

 3) What is T ½ of the drug / peak action?

 4) Mention 3 contraindications?

 5) Mention at least 6 adverse effects?
Answer for Question No : 27

1)   Act on the β1 adrenergic receptors of the

     myocardium.     It    increases    stroke   volume    –

     increased COP – causes peripheral vasodilatation –

     decreases     the    sympathetic    vascular   tone   –

     decreases the after load and there by improving

     the myocardial function.

2)   Dose 2.5 – 15mcg/kg/min. infusion rate of 6mg/kg

     in 100ml normal NS, 1ml/hr will give 1mcg/kg/min.

3)   T ½ - 2min, peak action 10 – 20min. ( Dobu2min ! )

4)   IHSS, atrial fibrillation and atrial flutter, sulfite

     sensitivity, hypotension.

5)   Increase myocardial O2 demand tachycardia,

     ectopic   heartbeat,     angina    /   palpitations   /

     tachyarrythmias, tingling sensation, parasthesia

     and leg cramps, diarrhoea and abdominal cramps.
Question No : 28

4 year old child is being evaluated for

syncope.

1) Identify the ECG?

2) Mention 2 acquired causes for the above

abnormality?

3) Drug of choice.

4) What should be taught to parents?
Answer for Question No : 28

1) Prolonged QT interval - > 0.45secds.

2) Myocarditis / electrolyte abnormality

     like calcium, mitral valve prolapse and

     drug induced.

3)   Beta adrenergic antagonist- β blockers

4) Parents           should   be      taught

     cardiopulmonary resuscitation.
Question No : 29

1) a) List the components of IMNCI?

b) List the components of reproductive and

child health programme?

2)   Mention   4   highlights   of   the   Indian

     adaptation of IMNCI?

3)Which        vaccine   project     has    been

     introduced as a part of pilot project in

     IMNCI?
Answer for Question No : 29
1a) Family planning

  Child survival and safe motherhood

  Client approach to health care

  Prevention and management of RTI /STD/ AIDS

b) Improvement in case management skills of health staff,

through provision of locally adopted guidelines and activities

to promote their care.

Improvement in overall health system

Improvement in family and community health care system.

2) Inclusion of 0-7 days age in the programme

   Incorporating national guidelines on malaria

  Anemia, Vit.A supplementation, and immunization schedules

  Training of the health personnel begins with sick young

  infants upto 2 months

  Proportion of training time devoted to sick young infant

  and sick child is almost equal

3) Hepatitis B vaccine



                  Question No : 30
1) What is the diagnosis?

2) What is the confirmatory test?

3) What     neurological   complications   can

   occur?

4) Mode of inheritance?

5) What antibiotic is prescribed for this

   as prophylaxis?
Answer for Question No : 30

1) Sickle cell anemia

2) Hemoglobin electrophoresis or HPLC

3) Stroke

4) Autosomal recessive

5) Penicillin
Question No : 31

These lesions are tender

 1) What is the diagnosis?

 2) What 2 common infections and drugs

     can trigger this?

 3) What 2 non – infectious systemic

     disorders can trigger this?
Answer for Question No : 31




1) Erythema Nodosum

2) TB, Streptococcus

     Sulfa, Phenytoin, Oral contraceptives

3)   IBD, Spondylo arthropathy, Sarcoidosis
Question No : 32

You are asked to provide prophylaxis for bacterial

endocarditis for 2 children with the following clinical

details.

 Child 1: 8 year old boy with rheumatic mitral

            regurgitation is to undergo dental extraction

            tomorrow.

       1) What is the drug of choice?

       2) Dosage and timing

Child 2: 2 years old male with VSD is to undergo

           Cystocopy tomorrow.

       1) What is the drug of choice?

       2) Dosage and timing.

Child 3 :3 year old who has undergone PDA ligation

           2 years back is to undergo dental extraction.

       What is the appropriate advice?

Child4: 2 year old with TOF is to undergo circumcision.

       What is the appropriate advice?
Answer for Question No : 32

1) Oral amoxycillin 50mg/kg 1hr before

   surgery         (or)

   Ampicillin IV/IM 50mg/kg ½ hr before

   surgery

2) IV Ampicillin 50mg/kg + gentamycin

   1.5mg/kg    30mins      before    surgery

   followed    6hrs     later   by   IV/oral

   Ampicillin/amoxycillin 25mg/kg

3) No prophylaxis needed

4) No need for anti microbial prophylaxis.
Question No : 33

Pedigree chart:




  1)   Identify the Mode of inheritance?

  2)   Give 3 examples of clinical disorders?

  3)   What is the significance of the pedigree symbols

       used in this?
Answer for Question No : 33




1) Autosomal dominant

2) Neurofibromatosis, Huntington’s

     chorea, Myotonic dystrophy.

3)       - Normal Male     - Normal female


         - Affected male   - Affected female


         - Proband         - Dead
Question No : 34

Pneumococcal 7 valent conjugate vaccine

1)   What is the protein conjugate?

2)   Route of administration?

3)   Youngest age for administration?

4)   Dosing interval?

5)   Primary immunization schedule for infants

     < 6 months of age?

6)   Dose if started at 12 – 23 months of age?

7)   Dose if started > 24 months to 9 yrs of

     age?
Answer for Question No : 34

1)   Diphtheria CRM 197 protein

2)   IM

3)   6 weeks

4)   4 to 8 weeks

5)   3 doses < 1 year

     1 dose 12 to 15 months

6)   2 doses

7)   1 dose
Question No : 35

     A study was carried out to assess the utility of IgM

Elisa test in the diagnosis of Leptospirosis. Blood culture

positive cases were considered the gold standard for

diagnosis.

     A total of 100 cases were studied.

     Leptospira were grown in blood culture in 40 of

these cases.

     IgM Elisa was positive in 70 out of 100 cases. Out of

these 70 cases, Leptospira were cultured in 30.

     IgM Elisa was negative in 30 cases, out of this 30,

Leptospira was grown in culture in 10 cases.

     Calculate the following for IgM Elisa as a diagnostic

test for Leptospirosis.

             1. Specificity

             2. Sensitivity

             3. Positive Predictive Value

             4. Negative Predictive Value
Answer for Question No : 35


                    Blood c/s    Blood c/s
                        +            -
      IgM elisa +   30 (a)       40 (b)       70
      IgM elisa -   10 (c)       20 (d)       30
                    40          60           100

1)   Specificity:
      d     x 100           20     x 100 = 33.3%
     d+b                 20 + 40
2)   Sensitivity:
      a     x 100           30    x 100 = 75%
     a+c                  30 + 10
3)   Positive predictive value:
       a     x 100         30     x 100 = 42.85%
      a+b                 30 + 40
4)   Negative Predictive value:
       d    x 100          20     x 100 = 66.6%
     c+d                 10 + 20




              Question No: 36
A 2 year old child presents with biphasic fever, severe

arthragia and rash tourniquet test is negative. Platelet

count is normal. Hb is 10.28gm.

  1)   What is the most probable diagnosis?

       a)   Dengue hemorrhagic fever

       b)   Measles

       c)   Chikunguniya fever

       d)   Roseola infantum

  2)   List 2 criteria for case definition of this probable

       case and4 criteria for confirmed case

  3)   What neurological complication can develop?

  4)   What family does this virus belong to?




       Answer for Question No : 36
1)   Chikunguniya

2)

     •    Features of suspect case-Fever with chills

          /     arthralgia    /    rash    /    rheumatic

          manifestations.

     •    Case definition – features of suspect case

          and positive serology in acute and convalesce

          phase.

     •    Confirmed case – probable case with any of

          the following :

     a)       4 fold rise in antibodies in paired sera

     b)       Positive IgM

     c)       Virus isolation from serum

     d)       Positive RT PCR in serum

     e)       Positive RT PCR in serum

3)   Meningo encephalitis

4)   Toga viridiae

                  Question No : 37
A 10 year old female weighing 30kg; diagnosed case

of IDDM on insulin therapy as follows:

                       10 (regular)
Morning: 40 units
                       30 (lente)
                       7 (regular)
Evening: 20 units
                      13 (lente)

Her recent morning blood sugars are becoming

high. (Blood sugar at 7.00 am         280 mg%)

  1)   What is Somogyi and Dawn phenomenon?

  2)   How will you differentiate these two in this

       case?

  3)   How will you treat in either case?


       Answer for Question No : 37
1)       Somogyi phenomenon: Hyperglycemia begetting

         hypoglycemia due to counter regulating hormones

         in response to insulin induced hypoglycemia.

         Dawn phenomen: Hyperglycemia (early morning)

         without preceding hypoglycemia due to decreased

         availability of insulin and increased GH release.

2)       Measure blood sugar at 3 am, 4 am, and 7 am.

     •   If blood sugar > 80mg/dl in 1st two sample, and

         high in 3rd Diagnosis – Dawn phenomenon

     •   If blood sugar <60mg/dl in first two sample and

         high in 3rd. Diagnosis – Somogyi phenomenon

3)       Treatment-

     • Dawn phenomenon – increased evening dose of

         Lente insulin by 10 – 15%

     • Somogyi phenomenon – decreased evening dose of

         Lente insulin by 10-15%.



                   Question No : 38
Councell the mother of a child who is being discharged

from your hospital following acute severe asthma.




     Answer for Question No : 38
1)   Introduces himself

2)   Clearly explains about asthma as hyperactive

     airway disease and not infective.

3)   Explains that there is no curative treatment and

     treatment reduces the severity and complications.

4)   Explains how to use MDI.

5)   Explains preventive strategies at home.

6)   Explains danger signs / warning signs of acute

     attack.

7)   Tells the treatment at home and reach nearest

     hospital. Tells difference between Rescue and

     prophylactic inhalers.

8)   Explains other alternatives, and ask for any

     doubts and clears it.

9)   Need for regular follow up.

10) Note of thanks and availability.
Question No : 39
1)   A child has massive GI bleed.

His clotting time is 12 mins. His PT test is 40 secs,

control 14secs and PTT test is 60 secs, control is 30

His Hb is 11gm and Platelet count is 2.5lakhs.

What blood product would you transfuse? And how

much?

2)   List 4 other indications for transfusion with this

     blood product?

3)   List 4 clotting factors deficiencies, which will be

     corrected by this transfusion?

4)   Name 2 parasitic disease transmitted by blood

     transfusion?
Answer for Question No : 39

1)   Fresh Frozen Plasma - 15ml/kg

2)   a) Severe clotting factor deficiency and

     bleeding

b) Severe clotting factor deficiency and invasive

procedure.

c)   Emergency reversal of warfarin effects

d)   Dilutional coagulopathy and bleeding.

e)   Anticoagulant protein (AT – III, protein C,

     Protein S, FTTP)

3)   II, V, X and XI

4)   Malaria / Chagas disease
Question No : 40

A 13 year old HIV positive boy is seen in OPD for

abrasions injuries over left thigh following RTA 2

days back. He was vaccinated with TT at age of 10

yrs.

       1)   What     immunization        advice   is

            appropriate?

       2)   Mention the dose and route?

       3)   What is the amount of Tetanus toxoid

            present in DPT, DT and TT?

       4)   What is the preservative in TT?

       5)   What is the role of Aluminum Phosphate

            in TT?
Answer for Question No : 40

1)   a) Clean the wound with soap and water.

     b) Administer TT & TIG

     c) Advice booster dose.

2)   a) TT 0.5ml IM on one buttock

     b) TIG 500U IM on opposite buttock

3)   10 Lf of TT component.

4)   Thiomerosol 0.01% w/v

5)   TT is adsorbed on to aluminum compounds

     and increases potency, by reaching high

     titers. It also gives long lasting immunity.
Question No : 41

An infant is being evaluated for ambiguous

genitalia. You find clitoral hypertrophy and other

signs of virilization. On investigations Sr. cortisol

levels are low. ACTH & PRA are markedly elevated.

ACTH skin stimulation test reveals markedly

increased    17     –   OH   progesterone.    Serum

testosterone is also elevated. Child also has severe

hyponatremia.

  1)   What is your diagnosis?

  2)   What is the mode of inheritance?

  3)   What is pre-natal diagnosis?

  4)   What advice will you give her for the next

       pregnancy?

  5)   What treatment you offer for this baby?
Answer for Question No: 41
1)   Congenital adrenal hyperplasia due to 21

     hydroxylase deficiency

2)   Autosomal recessive

3)   1sttrimesterCVS,2ndtrimester amniocentesis

     for DNA common mutations or polymorphic

     micro satellite markers if affected siblings

     samples are available for comparison.

4)   Dexamethasone      20mg/kg       pre   pregnancy

     maternal wt in two – three divided doses.

     Prefer CVS. continue treatment if female

     child.

5)   Hydrocortisone 10 to 20mg/m2/day tds.

     (Increase in stress situation)

Mineralo corticoids 0.1 – 0.3mg/day BD

Sodium supplements 1 – 3gm.

Surgical correction.
Question No : 42
Drug: Carbamzepine

1)   Mention 4 clinical indications?

2)   In which type of seizures it is avoided?

3)   What is therapeutic drug levels in blood and

     recommended time to draw sample?

4)   Mention 4 common drugs, which increases its

     toxicity?

5)   What dose adjustment is needed in ARF?

6)   What is the standard concentration in

     suspension?

7)   Mention dosage of frequency of suspension?

8)   What & how frequent lab monitoring is

     needed?

9)   Mention 4 life threatening complications?

10) What is treatment of toxicity?
Answer for Question No : 42
1)   GTCS,    partial   seizures,    trigeminal     neuralgia,

     bipolar disorders.

2)   Myoclonic seizures

3)   4 –12mg/L, 30min. before oral dose.

4)   Erythromycin, INH, TCA, clozapine, itraconazole,

     cimetidine.

5)   Mild, mod forms - dose adjustment is not

     necessary, severe (Creatnine clearance < 10;

     decrease dose by 75%)

6)   5ml = 100mg

7)   10-20mg/kg/day, increased 100mg/day at 1 week

     interval (BD/QD).

8)   CBC, SGOT, SGPT every monthly for first 3-4

     months and then as needed.

9)   Hyper    sensitivity   reactions,   aplastic     anemia,

     pancytopenia, hepatic toxicity, thrombocytopenia.

10) Gastric    lavage     repeated   dose   of      activated

     charcoal, hemoperfusion or hemodialysis.
Question

1 ½ year old male with acute gastroenteritis develops

anuria. His Hb is 8.9, Platelet count 90,000 & Pt –

24/20, PTT – 28/30. His PS is shown below.

  1)   Describe PS

  2)   Mention two MC D/D for it.

  3)   Which single test will help differentiate your

       D/B?

  4)   This baby’s renal functions de--erated after 2

       units of FFP. What would be the possibility?

  5)   What are indications of steroids in this

       scenario?

  6)   Which   other   conditions   here   similar    PS

       findings?
Answer

1)   Microangiopath--- hemolytic anemia with

     helmetcells, burn cells, he------- RBCs

2)   HUS, RVT

3)   Doppler USG -------

4)   HUS due to strep pneumonia

5)   Seizures

6)   Malignment HTN, SLE
Question

A 10 year old male with acute onset progressive lower

limb weakness. On detailed CNS examination you find

he     is    cons---   alert   and   normal   cranial   nerve

examinations. No bowel / bladder involvement. His

knee and ankele re--- are brisk and has grade II

power on both lower limbs. You also notice abdominal

reglesses below ----- is absent alibbus sensory system

normal.

  1)        what is your progressive diagnosis?

  2)        Mention    4   conditions    where    you     get

            hypotension and diminished referes in UMN

            lesion?

  3)        Mention 3 major points to differentiated extra

            modular from intramedially lesions?

  4)        Mention a congential conditions leading to non

            compressive myelopathy?
Question No : 43
A 8 year old boy is brought because his mother

feels he is short for his age. His height is 80cm.

His father’s height is 160cm and mother’s height is

148cm. His US/LS ratio is 1. 4: 1

  1)   What type of short stature does this child

       have?

  2)   What is the mid parenteral height of this

       child?

  3)   Name 3 causes for the short stature in this

       child?

  4)   What is the normal US/LS ratio at this age?

  5)   Name     3   conditions   in   which   there   is

       advanced US/LS ratio?
Answer for Question No : 43
1)   Dysproportionate dwarfism

2)   160cm

3)   Achandroplasia,   cretinism,   short    limb

     dwarfism.

4)   1.1:1

5)   Arachynodactyl,   chandrodystrophy,    spinal

     deformity and eunochodism
Question No : 44

An infant with seizures is being investigated. The

following are the lab reports.

Serum Calcium: 6.6mg%

Po4: 9mg%

SAP: 500 units

Mg: 3mg%

  1)   What is the probable diagnosis?

  2)   What will be the levels of PTH & 1, 25(OH2)

       D3?

  3)   The same infant is also noted to be dark and

       having mucocutaneous candidiasis.

       What is your diagnosis?

  4)   CT brain is carried out. What finding do you

       expect?
Answer for Question No : 44

1) Hypoparathyroidism

2) Both are low

3) Type     I   polyendocrinopathiy   (with

   Addison’s)

4) Basal ganglia calcification
Question No : 45




A   16   week   infant   is   examined    for

developmental   assessment.    Write     what

patterns of behaviour will you expect in this

age?
Answer for Question No : 45

Prone : Lifts head and chest, arms extended.

Ventral suspension : head above plane of body

Supine : TNR and reaches toward and misses

objects.

Sitting : No head lag on pulling, head steady,

tipped forward, enjoys sitting with truncal

support.

Standing : when held erect, pushes with feet.

Adaptive : Sees pellet, makes no move to it

Social : Laughs out loud, excited at sight of food,

may show displeasure if social contact is broken.
Question No : 46

A child with meningoenclphalitis is comatose. His

serum sodium is 116 and you are contemplating

diagnosis of SIADH

1)   Which of the following lab values will be

     present?

     a)   Urine OSM <100 mosm/l

     b)   Plasma volume normal or increased

     c)   Urine Na 200mg/L

     d)   Serum uric acid 10mg%

2)   Name 2 drugs which increases the vasopressin

     levels?

3)   The fluid intake should be restricted to

4)   Which drug may be given?

5)   2 acute respiratory illnesses, which cause this?
6)     Anticonvulsant,    which   decreases    ADH

       production?

       Answer for Question No : 46

     1) a) Urine OSM <100 mosm/l - Negative

       b)   Plasma volume normal or increased -

            Positive

       c)   Urine Na 200mEq/L - Positive

       d)   Serum uric acid 10mg% - Negative

2) CBZ, Vincristine, TCA

3)     1000ml/ m2 /24hr

4)     Demeclocycline

5)     Broncholitis, pneumonia

6)     Phenytoin
Question No : 47

1)   Identify this.

2)   What is the route of entry of this organism?

3)   Name 2 conditions that are high risk for this

     infection?

4)   What hematological clue will occur?

5)   Drug of choice?

6)   1 complication
Answer for Question No : 47

1)   Strongyloides stercoralis Larvae

2)   Skin

3)   HIV/ Immunosuppression, PEM, MR,

     Autoimmune disease

4) Eosinophilia

5)   Ivermectin 200mcg/kg OD for 1 – 2 days

6)   Hyperinfection syndrome
Question No : 48

A child with stroke is noted to have Ectopia Lentis,

arachinodactyly blue eyes and developmental delay.

  1)   What is the likely diagnosis?

  2)   What urine screening test will be positive?

  3)   Estimation of plasma amino acids will show

  ↑ cystine

  ↑ Methionine

  ↑ Homocystine

  4)   Which vitamin is indicated?

  5)   What other drug is needed?
6)   Mode of inheritance?




      Answer for Question No : 48

1)   Homocystinuria

2)   Positive cyanide nitroprusside test

3) ↑ cystine - Negative

     ↑ Methionine - Positive

     ↑ Homocystine - Positive

4)   B6 200 – 1000mg/24hr

5)   Betaine

6)   AR
Question : 49

You are to meet a child with Thalassemia major

1)   When do you start transfusion?

2)   Optimum Hb

3)   Intervals for transfusion

4)   What is the risk in keeping Hb > 14?

5)   When will HbF be low?

6)   How   do   you    reduce    non-hemolytic   febrile

     transfusion?

7)   What is the level of serum ferritin to be

     maintained?

8)   Indications for splenectomy
9)   When to give vaccines before splenectomy?

10) Antenatal diagnosis How and When?


      Answerfor Question No : 49

1)   < 7gm

2)   9 – 10gm

3)   2 – 4 weeks

4)   Thrombosis

5)   After repeated transfusions

6)   Leukocyte filter, pheneramine with paracetamol

7)   < 1000mg/L

8)   > 220ml/kg/yr of PRBCs, hypersplenism,

     massive    spleen   with   prominent     abdominal

     discomfort.

9)   Atleast 4 weeks
10)   CVS- mutation, 12to 14 weeks (MTP < 20weeks)




                Question No : 50

Match the disease with the Urine Screening Test

1) Galactosemia    Nitroprusside test

2) PKU                       CN PT

3) MPS                DNPH

4) HCU                Benedicts test

5) Organic aciduria      spot test

                       (Toludene blue test)

6) Cystinuria      Fecl2
Answer for Question No : 50

1) Galactosemia       Benedicts test

2) PKU                     Fecl2

3) MPS                MPS spot test

                        (Toludene blue test)

4) HCU                (Nitroprusside test)

5) Organic aciduria       DNPH

6) Cystinuria         CNPT
Question : 51

DRUG: Vigabatrine

1) Mode of action?

2) Dosage

3) Important side effects

4) Mention important uses
Answer for Question No : 51

1) Y    aminobutyric   acid,   transaminase

   inhibitor

2) 30mg/kg/d, od /bd upto 100mg/kg/d

3) Visual field constriction, Optic atrophy,

   optic neuritis

4) Infantile spasms, Tuberous sclerosis

   adjuvant for poorly controlled seizures.
Question No : 52




A 2 year old child is brought for Toe walking.

 1) What is the commonest cause?

 2) What is the Differential diagnosis?

 3) Upto what age is it normal?
Answer for Question No: 52

1) Normal children

2) Cerebral   palsy,   Duchenne    muscular

   dystrophy, tethered cord, congenital

   tendo-achilles contracture, leg-length

   discrepancy, CDH and habitual

3) 3 year
Question No : 53

1) What is premature thelarche?

2) Upto what age is it benign?

3) What    is   exaggerated      or     atypical

   thelarche?

4) What will be the level of FSH, LH,

   oestradiol    in   benign          premature

   thelarche?

5) What will be the USG findings?
Answer for Question No : 53

1) Isolated breast development

2)   < 3 yrs

3) Associated with accelerated bone age

     due to systemic ostrogen effects

4) Low

5) Small Ovanian cysts
Question No: 54

A child has chronic polyarthritis of 4 joints and is

ANA positive.

  2)   What arthritis is it likely to be?

  3)   What complication should we anticipate?

  4)   In which type of JRA is HLA β27 positive ?

  5)   A child with JRA presents with fever,

       leuepenia   and    hepatosplenomegaly     and

       lymphadenopathy. What is the diagnosis?

  6)   What drug is indicated in treatment of Q4?
Answer for Question No: 54

1)   JIA pauciarticular type I

2) Chronic uveitis

3)   Pauciarticular type II

4) MAS

5) Cyclosporin
Question No :55

Oxygen therapy

 1)   Below what blood O2 does WHO recommend

      O2 therapy?

 2)   What are the clinical indications for O2

      therapy?

 3)   O2 concentration with reference to FiO2

      Nasal prongs @1-2 lit/min, Nasopharyngeal

      catheter
Answer for Question No : 55

1)   <90%

2)   Central cyanosis, unable to drink due to

     respiratory distress.

      In those with pneumonia, Broncholitis and

asthma- severe lower chest in drawing, RR >70,

Grunting, Head nodding

3)   FiO2 30 – 35 %, 45 – 60 %
Question No : 56

A child is brought with snakebite.

1)   In which of the following is appropriate as

     first aid.

     a)   Splint the limb

     b)   Apply ice

     c)   Apply tourniquet to occlude venous flow

     d)   Clean the wound

     e)   Transport to hospital

2)   2 specific indication for ASV

3)   What is the dose of ASV to a 3 year old?

4)   What is the diluent for ASV?
Answer for Question No : 56

1) a)     Splint the limb -

     b)   Clean the wound –

     c) Transport to hospital –

2) Indications:

 a) Systemic signs of envenomation

b) Local symptoms like severe necrosis, swelling

of > half of the limb

3) Same as adult

4)   Normal saline (2 to 3 volumes)
Question No: 59




1) What are the 4 types of lesions?

2) What bacteria cause this?

3) Which drug can induce this?

4) Name 4 drugs used?

5) What Dietary advice will you give?
Answer for Question No : 59

  1)   Open Comedones – blackhead

       Closed comedones – whitehead

       Papules, pustules and nodulocystic lesions

  2)   Propionibacterium acnes

  3) Corticosteroid, androgens, INH, phenobarbital,

  Phenytoin, B12 and lithium

  4) Benzoyl Peroxide, Tretinoin, Adapalene,

Topical – Erythromycin and clindamycin

Use for 4 to 8 weeks:

Systemic therapy: Tetracycline, Doxy, Minocycline

Isotretinoin (nodulocystic) (teratogenic)

Intradermal triamcinolone

   5) Normal
Question No : 57

A child presents with muscle cramps. The serum

Magnesium is 1mg/dl, K is 1.6mg/dl and Hco3 is 40mg/dl.

There is no dehydration.

  1)   What electrolyte in urine will you estimate?

  2)   Why?

  3)   The level of the urinary electrolyte estimated is

       high – BP is normal. List 3 possible diagnosis?

  4)   There is no history of drug ingestion or failure to

       thrive hypertension. What is the diagnosis?

  5)   3 drugs for treatment

  6)   What will be level of renin and aldosterone in

       serum?

  7)   What will be the urinary calcium level?
Answer for Question No : 57

1)   Calcium

2)   To distinguish low and high urine calcium

     levels

3)   Barter’s, giltelman’s and base administration

4)   Giltelman syndrome

5)   Na, mg and spiranolactone

6)   Normal

7)   Low
Question No : 58

A child is admitted with TCA poisoning.

  1)   What are the 3 ‘C’s in manifestations?

  2)   What ECG findings do you anticipate? (3)

  3)   Which of the following is correct?

       a)   Emesis is indicated

       b)   Activated charcoal to be given

       c)   Na Hco3 must

       d)   PH should be 7.45 to 7.55

       e)   Lidocaine not be used for any time

       f)   Quinidine & procainamide - to be used

       g)   NaHco3 is used to prevent cardiac

            arrhythmias
Answer for Question No : 58

1)   Coma, convulsion, cardiac toxicity

2) Widening of QRS, Q-T prolongation,-, flat

     or inverted T , ST depression, RBB, CHB

3) A) No

     b) Yes

     c)   Yes

     d) Yes

     e) No

     f)   No

     g) Yes
Question No : 61




1)   What is the diagnosis?

2) Name 2 topical agents of use?

3) Duration and frequency?

4) 2 drugs for systemic therapy?

5) Commonest organisms?
Answer for Question No : 61

1)   Tinea corporis

2) Miconazole,     ketoconazole,   clotrimazole,

     econazole, terbinafine, niftifine

3) Bd, 2 to 4 week

4) Griseofulvin – several weeks

Itraconazole – 1 – 2 week

5) T. Rubrum, T. Mentagrophytes
Question No : 60

Write the calorie value of
Rice – 1 cup

Puri – 1

Upma – 1 cup

Idli – 1

Dosa – 1

Kichidi - 1 cup

Boiled egg - 1

Vada - 1

Pizza – 1 slice

Oil – 1 tbsp

Ice cream – ½ cup

Peanuts – 50 nos

Banana – 1

Cashew nuts – 10

Milk chocolate – 25gm
Answer for Question No : 60
Rice – 1 cup - 170

Puri – 1       - 100

Upma – 1 cup - 270

Idlli – 1      - 75

Dosa – 1       - 125

Kichidi - 1 cup - 200

Boiled egg – 1 - 90

Vada – 1       - 70

Pizza – 1 slice - 200

Oil – 1 tbsp   - 60

Ice cream – ½ cup - 200

Peanuts – 50 nos - 90

Banana – 1     - 90

Cashew nuts – 10 - 95

Milk chocolate – 25gm - 140
Question No : 62

This girl has palpitation, diarrhea, and loss of

weight

1)   What is the diagnosis?

2)   The thyroid swelling is not tender, not

     nodular. She has exophthalmos. What is the

     cause?

3)   List 4 other causes?

4)   Investigations

5)   2 drugs used
Answer for Question No : 62

1)   Hyperthyroidism

2)   Graves Disease

3)   Toxic adenoma, toxic multinodular , subacute

     thyroiditis, Lymphocytic thyroiditis, iodine

     induce,   exogenous     hormone,    pituitary

     adenoma and ovarian tumor.

4)   TSH is decreased, T4 increased. Thyroid

     uptake increased

5)   β blockers, methimazole (propylthiouracil)
Question No : 63

A spirometry is performed in an asthmatic child

  1)   What will be abnormalities in the following:

  FEV1

  FEV1 / FVC

  Improvement in FEV1

  Exercise challenge

  2)   What PEFR variation that is consistent with

       a diagnosis of asthma?
Answer for Question No : 63

1)   FEV1 – Low

FEV1 / FVC ration < 0.8

Improvement in FEV1 with inhaled β2 agonist

≥12%

Exercise challenge – worsening in FEV1 ≥ 15%

2) Morning to afternoon variation ≥ 20%
Question No : 64

The following is the ABG is an infant

PH - 7.2, Hco3 – 10, CO2 – 30

1)   What is the diagnosis?

2)   Why?

3)   Give an example of a common clinical setting

     for this condition?

4)   Write the other compensation in acid bone

     disorder. Which are appropriate?
Answer for Question No : 64

1)    Metabolic acidosis – respiratory acidosis

2)    Expected Co2 = 1.5 X (10) + 8 ± 2

                      = 21 to 25

     Co2 >25

3)    Pneumonia with sepsis (lactic acidosis and respiratory

      acidosis)

4)    Metabolic alkalosis Pco2 ↑ 7mm for 10meq/L of Hco3

      Respiratory acidosis – Acute: Hco3 ↑ 1 for 10mm ↑ in

      Pco2

      Chronic: Hco3 ↑ 3.5 for 10mm ↑ in Pco2

      Respiratory alkalosis Acute: Hco3 ↓ 2 for 10mm ↓

      in Pco2

      Chronic: Hco3 ↓ 4 for 10mm↓ in Pco2
Question No : 65

Serum Na - 136, Cl - 102 and Hco3 - 10

1)    What is the anion gap?

2)    What is the normal anion gap?

3)    In which of the following is anion gap normal or

      increased?

Diarrhea

Lactic acidosis

DKA

ARF

RTA

Salicylate poisoning

Urinary tract diversion

IEM

Septic shock

Post hypocapnea
Answer for Question No : 65

1)   (136) – (102 + 10) = 24

2)   8 – 16

3) Diarrhea - Normal

Lactic acidosis - increased

DKA - increased

ARF - increased

RTA - Normal

Salicylate - increased

Urinary tract diversion - Normal

IEM - increased

Septic shock - increased

Post hypocapnea - Normal
Question No : 66




1)   What is the mode of inheritance?

2)   2 examples

3)   Characteristics of the inheritance

4)   Plasma ammonia in this child is normal. What

     is the likely diagnosis?

5)   There is no odor or skin lesion, but there is

     ketosis. What is the likely diagnosis?
Answer for Question No : 66

1)     XLD

2)     VDRR         (Hypophosphotemia),       incontinentia

       pigmenti

3)     Affected men          all    affected

                             All     normal


     All        +    of affected have 50% inheritance

     Rare XLD -         milder disease


                       Thrice as common as male

4) Organic acidemia

 Urea cycle – NH3 , anion gap – normal

Amino acid defects

           or

Galactosemia              NH3 – normal, anion gap – normal
5) MMA,

    Propionic acidemia,

     Ketothiolase deficiency

Odor + - MSUD / isovaleric

Skin + - Multiple carboxylase deficiency

Ketosis - Acyl CoA, 3Hydroxy 3HGA, HMG co -

synthetase deficiency
Question No : 67

1)   What is the clinical classification of leprosy

     in India?

2)   What    are     the   differences   in   leprosy

     constitution?

3)   What is the WHO recommended standard

     treatment regime for children aged 10 – 14

     yrs?
Answer for Question No : 67

1)    Indeterminate

      Tuberculoid      Paucibacillary 1 to 5 lesions

      Borderline        in skin

      Lepromatous         Multi bacillary > 5 lesions

      Pure neuritis

     2) Case of leprosy – clinical sign +


                         Bacilli in smear +

                         Not completed treatment

Paucibacillary – 1 to 5 lesions

Multibacillary

Adequate treatment

Repeated treatment
Newly diagnosed case

Defaulter

Relapsed case

3) Multi Bacillary (completed in 12 months)

   RMP 450mg once a month

   Clofazamine 150mg once a month (supervised)

  Clofazamine 50mgEod (self administered)

  Dapsone 5omg once a month (supervised)

  Dapsone 50mg daily dose ( domiciliary)

Pauci Bacillary (completed in 6 months)

 Rifampicin 450mg once a month

Dapsone 50mg OD daily (domiciliary)
Question No : 68

Midday meal program

1) What are the principles?

2) Write a model menu?
Answer for Question No : 68

1)    Meal – a supplement, not a substitute to

      home diet

Should supply 1/3 of total energy, ½ of protein

Cost low

Easily cookable in school

Use locally available foods

Change menu frequently

2)    Cereals & and millets – 75gms

     Pulses - 30

     Oil & fat - 8

     Leafy vegetable –30

     Non-leafy - 30
Question No : 69

Answer the following questions with regard to

cold chain equipment:

1)   Which vaccines are stored in deep freezers?

2)   Which vaccines are stored in ILR?

3)   Which vaccines are kept in the basket of

     ILR?

4)   What is the function of cold boxes?

5)   What are Day carriers used for?

6)   What are vaccine carriers used for?

7)   Which vaccines should not be frozen?
Answer for Question No : 69

1)   Measles & OPV

2)   All

3)   TT, DPT, DT and diluents (not in the floor –

     may be frozen)

4)   Transportation of vaccines

5)   Carry small quantities of vaccines to a nearly

     session.

6)   Carry small quantities of vaccines to out of

     reach session

7)   DPT, DT, TT, Typhoid, BCG, HBV, diluent
Question No :

A 11 year old boy is brought with a penetrating crush

injury with a compound fracture. His immunity and

immunization status for tetanus is unknown. Which of the

following action is correct with regard to tetanus

prevention?

       a)     Nothing is required

       b)     Toxoid 1 dose

       c)     Toxoid 1 dose + TIG

       d)     Toxoid complete course + TIG

His 8 year old sister has multiple clean abrasions. She

has earlier received 3 doses of DPT in the first year and

1 booster at 1 ½ year and no other vaccines after that

What prevention will you carry out?
Answer for Question No :

  All wounds – surgical t------          < 6hr, clean, non

  penetrating with negligible tissue damage

       a)   Nothing

       b)   TT 1

       c)   TT 1

       d)   TT complete course

All wounds – surgical t------ other wounds       immunity

category.

       a)   Nothing

       b)   TT 1

       c)   TT 1 + TIG

       d)   TT complete course + TIG

       a)   Complete course + Bon----- < 5 yrs

       b)   Complete course + 5 to 10 yrs

       c)   Complete course + > 10 yrs

       d)   Immunity unknown + has not heel a complete

            course of toxoid.
Question No : 70
The peak flow rates of 10 children of same age are

as follows:

250, 260, 290, 200, 240, 240, 260, 270, 270, 290

  1)   What is the range?

  2)   What is the mean deviation and Mean?

  3)   What is the standard deviation?
Answer for Question No : 70
  1)    90 (200 to 290)

  2)    Mean Deviation =∑ (x – x)

                              N

  Mean – 257, Mean Deviation – 19.8

  3) Standard deviation = ∑ ( x – x ) 2       (>30)

                                    n

       Standard Deviation = ∑ ( x – x ) 2     (<30)

                                  n-1 in this case (10-1)

Take the deviation of each value from Mean (x-x)

  Square each (x – x )2

  Add and squared deviation ∑ ( x – x ) 2

  Divide by no. of observation or n-1 if <30

  Then take square root
Answer No for Question No : 71
  Resected tissue from OT Chemical disinfect ion

                             and discharge into drain

  Waste from laboratory      Chemical disinfect ion

   culture                   / autoclave /

                             microwave and mutilation

                             shredding

  Needles and syringes      Disinfect ion / shredding

  Discarded medicines      Autoclaving / microwaving

                           / incineration

 Linen contaminated        Incineration autoclaving

 with Blood               / Microwaving

 Used IV set              Incineration and drug

                          disposal in secured land

                          fills

Liquid waste from house   Incineration / deep burial

keeping
Question No : 71
  Write the correct method of treatment and disposable

  of the following categories of biomedical work?

Resected tissue from OT    Incineration / deep burial

Waste from laboratory      Autoclaving / microwaving

culture                    / incineration

Needles and syringes      Disinfect ion / shredding

Discarded medicines       Incineration and drug

                          disposal in secured land

                          fills

Linen contaminated        Incineration, autoclaving

with Blood                / Microwaving
Used IV set                 Chemical disinfect ion /

                          Autoclaving / microwaving/

                          Multilation/ shredding

Liquid wask from house    Chemical disinfect ion

keeping                    and discharge into drain
Question No : 72



This child also has joint hypermobility

  1)   What is the diagnosis?

  2)   What is the usual mode of inheritance?

  3)   What is the defect?

  4)   How many clinical forms?

  5)   What cardiac among can occur?

  6)   What surgical emergencies?

  7)   Difference with Cutis Laxa
Answer for Question No: 72

  1)   Ehlers Danlos

  2)   AD

  3)   Defect of fibrillar collagen – quantitative

  4)   10

  5)   MVP, AR

  6)   Rupture of great vessels, dissecting aneurysm,

       stroke,    rupture   of     uterus   in   pregnancy,

       echymoses, periodantitis.

  7)   Cutis Laxa – skin hangs in redundant folds – AR

  EDS – hyperextensible snaps back into place when

  stretched - AD

Cutis Laxa – Bloodhound appearance, aged appearance

Hyperelasticity and hypermobility of joints, hoarse cry,

lax vocal cords
Question No : 73
A child is brought with a history of accidental

ingestion of Iron tablets.

  1)   Which one of the following would be of

       benefit?

  Gastric Lavage

  Activated charcoal

  Whole bowel irrigation

  2)   When would you measure serum Iron?

  3)   What is the level of serum Iron which

       indicate significant toxicity?

  4)   If serum iron level reports were delayed,

       how would you confirm iron ingestion?

  5)   What are the 2 indications for giving

       desferioxime?

  6)   What system exhibits symptom first?

       When?
Answer for Question No : 73

1)   Whole bowel irrigation

2)   4 – 8 hrs after ingestion

3)   >500µg/dl

4)   X – ray abdomen

5)   Level >500µg/dl

     Moderate to severe symptoms

6)   GI, 30 minutes to 6 hr
Question No : 74




1)   What is the diagnosis?

2)   What 2 findings are characteristics?

3)   What is the effect of treatment?

4)   List 4 drugs useful

5)   What physical therapy will help?
Answer for Question No : 74
1)   Psoriasis

2)   A) Plaques with yellowish white scale like

     mica

b) Auspitz sign – pinpoint bleed or removal

3)   Koebner phenomenon lesions appear

4)   Coal tar, topical steroid, salicylic acid,

     calcipotriene (Vit.D analog), Methotrexate &

     cyclosporine and retinoid

5)   UV light
Question No : 75

Assess the development of this 3 year

old
Answer for Question No : 75

Motor : Rides tricycle, stands on the foot

momentarily

Adaptive : Tower of 10 cubes, imitates bridge

construction of 3 cubes, copies a circle, imitates

a cross

Language : Knows age and sex, counts 3 objects

correctly, repeats 3 numbers or a sentence of 6

syllables

Social : Plays simple games with other children in

parallel, helps in dressing – put on shoes,

unbuttons, washes hands.
Question No : 76




1) What is the diagnosis?

2) Commonest organism?

3) Treatment
Answer for Question No : 76
1)   Cutaneous larva migrans

2)   A. Braziliense (Hook worm of dogs and cats)

(other anky + & Strongyloides)

3)   Ivermectin – 200mg/kg/one 1 to 2 days

     Albendazole – 1 OD X 3 days

     Topical thiabendazole
Question No :

A mother says she has the following problems in breast-

feeding

  1)   A not enough milk

  2)   The baby is reluctant to breast feed
Answer for Question No :
If not enough milk – poor weight ----- <500/---

                                        <125gm/---

                            < birth weight after 2 weeks

<6 times / day urine strong / smelling concentrated urine

Common reasons :

Poor breast feed p------- :

      Poor attachment, no night feeds, delayed start, short feeds,

rigid schedule, broke, other feeds

Psychological – strem, tired

Physical causes

Baby is illness / con---- among

Advice :

Refusal or reluctance to breast feed :

Baby is in pain, ill, sedation

Encourage to ---- and feed more often

Use EBM

Rooming in

Correct positioning

Clear ------ nose

Treat oral

↓ sedation to mother
Question No : 77

Amniocentesis is brief contemplated for pregnant

woman for genetic counseling

  1)   What is the ideal time?

  2)   What is the most common indication?

  3)   Name 4 other indications?
Answer for Question No : 77

1)   15 to 16 weeks

2)   Advanced maternal age > 35 yrs

3)   a) Previous child: chromosomal anmaly

     b) Either parent - a translocation cause

     c) History of genetic disorder diagnosed

       by DNA analysis / biochemistry

     d) Sex detection in XLD / XLR diseases

     e) Maternal blood testing (triple screening)

     indication   risk

     f) Work up for fetal anomalies suggested by

     USG
Question No : 79




1)   Identify the abnormality in RBC?

2)   This child has chronic diarrhea. What is

     the diagnosis?

3)   Which vitamin deficiency in these children is

     associated with neurological symptom?

4)   Which lipid abnormalities are

     characteristics?

5)   What is the fundus finding?

6)   What is the mode of inheritance?


     Answer for Question No : 79
1)   Acanthocytosis

2)   A Betalipoprotenemia

3)   Vit. E

4)     Cholesterol

      TGL

     Absent B Liproteins

5)   Retinitis pigmentosa

6)   Autosomal recessive




              Question No : 78
An infant has cough and difficult breathing

The respiratory rate 70 / min

The infant has severe respiratory distress (head

nodding)

  1)   What does the infant have as per ARI

       programme?

  2)   What are other criteria for this status?

  3)   Will you treat this infant as OP or IP?

  4)   What is the antibiotic therapy regimen?




       Answer for Question No : 78

  1)   Very severe pneumonia
2)   Central cyanosis

     -------- to feed / drunk or vomited

     everything

     --------- / lethargy / -----------

3)   IP

4)   Ampi + Gent—for 5 days

     Oral ------- for 5 days

            Or

     CM for 10 days

            Or

     Ceftriaxone


              Question No: 80
1)   Identify the organism?

2)   What 4 stains are used?

3)   What is the treatment?
Answer for Question No : 80
1)   Pneumocystis carinii

2)   Grocott- Gomori      cyst

     Toluidine blue

     Polychrome – Giemsa

     Trophozoites and sporozoites

     Fluorescent labeled MAB

3)   5mg/kg once daily 3 day a week

     Cotrimoxazole

     15 – 20mg in 4

     3 weeks for AIDS

     2 weeks for others

     Pentamidine
Atovaquone, trimextrate + steroids

Answer for Question No . 82

  1)    Giandia Lambia

  2)    Acute Explosive fowl smelling watering
        diarrhoea
        Abd distusion / flatuluce / nansea anorara
        and epigastic cramps

  3)    FTT / Lactose Mal Absorbtion / Persistant
        Steattorrhoea, E hystlylica diarrhoea
        injection
  4)    Metronidayole 15 kg

Más contenido relacionado

La actualidad más candente

Osce in pediatrics
Osce in pediatricsOsce in pediatrics
Osce in pediatricsAli Shuaib
 
Pediatrics OSCE pictures
Pediatrics OSCE pictures Pediatrics OSCE pictures
Pediatrics OSCE pictures Shahd Al Ali
 
Mock OSCE in Pediatrics Apr 2014 Part 1 qn ans
Mock OSCE in Pediatrics Apr 2014 Part 1 qn ansMock OSCE in Pediatrics Apr 2014 Part 1 qn ans
Mock OSCE in Pediatrics Apr 2014 Part 1 qn ansDr Padmesh Vadakepat
 
DNB Pediatrics OSCE CME (Command Hospital, Pune)
DNB Pediatrics OSCE CME (Command Hospital, Pune)DNB Pediatrics OSCE CME (Command Hospital, Pune)
DNB Pediatrics OSCE CME (Command Hospital, Pune)Dr Padmesh Vadakepat
 
Practical pediatric quiz - Kaun Banega Winner
Practical pediatric quiz - Kaun Banega WinnerPractical pediatric quiz - Kaun Banega Winner
Practical pediatric quiz - Kaun Banega WinnerGaurav Gupta
 
OSCE MAY 2022-PART-4 -PAED.pptx
OSCE MAY 2022-PART-4 -PAED.pptxOSCE MAY 2022-PART-4 -PAED.pptx
OSCE MAY 2022-PART-4 -PAED.pptxGururajaRamaiah1
 
Chronic Liver Disease in pediatric: a case presentation and discussion
Chronic Liver Disease in pediatric: a case presentation and discussionChronic Liver Disease in pediatric: a case presentation and discussion
Chronic Liver Disease in pediatric: a case presentation and discussionDr Abdalla M. Gamal
 
Pediatrics OSCE pictures part 2
Pediatrics OSCE pictures part 2 Pediatrics OSCE pictures part 2
Pediatrics OSCE pictures part 2 Shahd Al Ali
 
Paediatrics quiz
Paediatrics quizPaediatrics quiz
Paediatrics quizsai raman
 
Pediatric neurology mcq
Pediatric neurology mcqPediatric neurology mcq
Pediatric neurology mcqHari Meshram
 
Dnb pediatrics osce 2 for PGS in Southern Railway Hospital
Dnb pediatrics osce 2 for PGS in Southern Railway HospitalDnb pediatrics osce 2 for PGS in Southern Railway Hospital
Dnb pediatrics osce 2 for PGS in Southern Railway HospitalNibedita Mitra
 
OSCE Pediatrics CME (Dr.D.Y.Patil Medical College)
OSCE Pediatrics CME (Dr.D.Y.Patil Medical College)OSCE Pediatrics CME (Dr.D.Y.Patil Medical College)
OSCE Pediatrics CME (Dr.D.Y.Patil Medical College)Dr Padmesh Vadakepat
 
OSCE Pediatrics Observed Stations Dr.D.Y.Patil Medical College CME
OSCE Pediatrics Observed Stations Dr.D.Y.Patil Medical College CMEOSCE Pediatrics Observed Stations Dr.D.Y.Patil Medical College CME
OSCE Pediatrics Observed Stations Dr.D.Y.Patil Medical College CMEDr Padmesh Vadakepat
 
Slide show for paediatric trainees
Slide show for paediatric traineesSlide show for paediatric trainees
Slide show for paediatric traineesVarsha Shah
 

La actualidad más candente (20)

Pediatrics OSCE, RIPE 2013
Pediatrics OSCE, RIPE 2013 Pediatrics OSCE, RIPE 2013
Pediatrics OSCE, RIPE 2013
 
Mock OSCE Pediatrics Apr 2013
Mock OSCE Pediatrics Apr 2013Mock OSCE Pediatrics Apr 2013
Mock OSCE Pediatrics Apr 2013
 
Osce in pediatrics
Osce in pediatricsOsce in pediatrics
Osce in pediatrics
 
Pediatrics OSCE pictures
Pediatrics OSCE pictures Pediatrics OSCE pictures
Pediatrics OSCE pictures
 
Mock OSCE in Pediatrics Apr 2014 Part 1 qn ans
Mock OSCE in Pediatrics Apr 2014 Part 1 qn ansMock OSCE in Pediatrics Apr 2014 Part 1 qn ans
Mock OSCE in Pediatrics Apr 2014 Part 1 qn ans
 
DNB OSCE SGRH - 2
DNB OSCE SGRH - 2DNB OSCE SGRH - 2
DNB OSCE SGRH - 2
 
DNB Pediatrics OSCE CME (Command Hospital, Pune)
DNB Pediatrics OSCE CME (Command Hospital, Pune)DNB Pediatrics OSCE CME (Command Hospital, Pune)
DNB Pediatrics OSCE CME (Command Hospital, Pune)
 
Practical pediatric quiz - Kaun Banega Winner
Practical pediatric quiz - Kaun Banega WinnerPractical pediatric quiz - Kaun Banega Winner
Practical pediatric quiz - Kaun Banega Winner
 
OSCE MAY 2022-PART-4 -PAED.pptx
OSCE MAY 2022-PART-4 -PAED.pptxOSCE MAY 2022-PART-4 -PAED.pptx
OSCE MAY 2022-PART-4 -PAED.pptx
 
Chronic Liver Disease in pediatric: a case presentation and discussion
Chronic Liver Disease in pediatric: a case presentation and discussionChronic Liver Disease in pediatric: a case presentation and discussion
Chronic Liver Disease in pediatric: a case presentation and discussion
 
Pediatrics OSCE pictures part 2
Pediatrics OSCE pictures part 2 Pediatrics OSCE pictures part 2
Pediatrics OSCE pictures part 2
 
Paediatrics quiz
Paediatrics quizPaediatrics quiz
Paediatrics quiz
 
approach to comatose child
approach to comatose childapproach to comatose child
approach to comatose child
 
Pediatric neurology mcq
Pediatric neurology mcqPediatric neurology mcq
Pediatric neurology mcq
 
Dnb pediatrics osce 2 for PGS in Southern Railway Hospital
Dnb pediatrics osce 2 for PGS in Southern Railway HospitalDnb pediatrics osce 2 for PGS in Southern Railway Hospital
Dnb pediatrics osce 2 for PGS in Southern Railway Hospital
 
OSCE Pediatrics CME (Dr.D.Y.Patil Medical College)
OSCE Pediatrics CME (Dr.D.Y.Patil Medical College)OSCE Pediatrics CME (Dr.D.Y.Patil Medical College)
OSCE Pediatrics CME (Dr.D.Y.Patil Medical College)
 
OSCE Pediatrics Observed Stations Dr.D.Y.Patil Medical College CME
OSCE Pediatrics Observed Stations Dr.D.Y.Patil Medical College CMEOSCE Pediatrics Observed Stations Dr.D.Y.Patil Medical College CME
OSCE Pediatrics Observed Stations Dr.D.Y.Patil Medical College CME
 
Slide show for paediatric trainees
Slide show for paediatric traineesSlide show for paediatric trainees
Slide show for paediatric trainees
 
Shock in children
Shock in childrenShock in children
Shock in children
 
OSCE - Pune mock OSCE 2012
OSCE - Pune mock OSCE 2012OSCE - Pune mock OSCE 2012
OSCE - Pune mock OSCE 2012
 

Destacado

OSCE Pediatrics Observed Stations (Mock Exam Apr 2013)
OSCE Pediatrics Observed Stations (Mock Exam Apr 2013)OSCE Pediatrics Observed Stations (Mock Exam Apr 2013)
OSCE Pediatrics Observed Stations (Mock Exam Apr 2013)Dr Padmesh Vadakepat
 
Pediatrics history taking
Pediatrics history takingPediatrics history taking
Pediatrics history takingRamzan Ali
 
Core clinical cases in pediatrics
Core clinical cases in pediatricsCore clinical cases in pediatrics
Core clinical cases in pediatricskanyaw
 
Paediatrics - Case presentation: fever+rash
Paediatrics - Case presentation: fever+rashPaediatrics - Case presentation: fever+rash
Paediatrics - Case presentation: fever+rashpatrickcouret
 

Destacado (6)

OSCE Pediatrics Observed Stations (Mock Exam Apr 2013)
OSCE Pediatrics Observed Stations (Mock Exam Apr 2013)OSCE Pediatrics Observed Stations (Mock Exam Apr 2013)
OSCE Pediatrics Observed Stations (Mock Exam Apr 2013)
 
Febrile seizures
Febrile seizuresFebrile seizures
Febrile seizures
 
Pediatrics history taking
Pediatrics history takingPediatrics history taking
Pediatrics history taking
 
Febrile seizures
Febrile seizuresFebrile seizures
Febrile seizures
 
Core clinical cases in pediatrics
Core clinical cases in pediatricsCore clinical cases in pediatrics
Core clinical cases in pediatrics
 
Paediatrics - Case presentation: fever+rash
Paediatrics - Case presentation: fever+rashPaediatrics - Case presentation: fever+rash
Paediatrics - Case presentation: fever+rash
 

Similar a OSCE Pediatrics KKCTH

Similar a OSCE Pediatrics KKCTH (20)

OSCE KKCTH 2007
OSCE KKCTH 2007OSCE KKCTH 2007
OSCE KKCTH 2007
 
Segundo simulador primera parte
Segundo simulador primera parteSegundo simulador primera parte
Segundo simulador primera parte
 
DNB Pediatrics OSCE June 2013
DNB Pediatrics OSCE June 2013DNB Pediatrics OSCE June 2013
DNB Pediatrics OSCE June 2013
 
Vars
VarsVars
Vars
 
Ospe 21 march 2017
Ospe 21 march 2017Ospe 21 march 2017
Ospe 21 march 2017
 
Osce cuckoos
Osce cuckoosOsce cuckoos
Osce cuckoos
 
Bronchial asthma pediatric
Bronchial asthma pediatricBronchial asthma pediatric
Bronchial asthma pediatric
 
Neonatology test
Neonatology testNeonatology test
Neonatology test
 
Aiims PGE-2004
Aiims PGE-2004Aiims PGE-2004
Aiims PGE-2004
 
Aiims questions with answers nov -2004
Aiims questions with  answers nov -2004Aiims questions with  answers nov -2004
Aiims questions with answers nov -2004
 
2011 inpatient updates participant slides
2011 inpatient  updates participant slides2011 inpatient  updates participant slides
2011 inpatient updates participant slides
 
Pediatric neurology notes
Pediatric neurology notesPediatric neurology notes
Pediatric neurology notes
 
Nephrotic syndrome.
Nephrotic syndrome.Nephrotic syndrome.
Nephrotic syndrome.
 
Business studies
Business studiesBusiness studies
Business studies
 
Quiz on pharmacology
Quiz on pharmacologyQuiz on pharmacology
Quiz on pharmacology
 
Quiz final-pharmacy
Quiz final-pharmacyQuiz final-pharmacy
Quiz final-pharmacy
 
QuizNote It is recommended that you save your response as y.docx
QuizNote It is recommended that you save your response as y.docxQuizNote It is recommended that you save your response as y.docx
QuizNote It is recommended that you save your response as y.docx
 
PICU OSCE.pdf
PICU OSCE.pdfPICU OSCE.pdf
PICU OSCE.pdf
 
MaximoGuerrieri_Bio_Mac
MaximoGuerrieri_Bio_MacMaximoGuerrieri_Bio_Mac
MaximoGuerrieri_Bio_Mac
 
Ospe of eye
Ospe of eyeOspe of eye
Ospe of eye
 

Más de Dr Padmesh Vadakepat

Neonatal Nursing of Extremely Premature Neonates - Dr Padmesh
Neonatal Nursing of Extremely Premature Neonates - Dr PadmeshNeonatal Nursing of Extremely Premature Neonates - Dr Padmesh
Neonatal Nursing of Extremely Premature Neonates - Dr PadmeshDr Padmesh Vadakepat
 
Update on Antenatal Steroids 2021 - Dr Padmesh
Update on Antenatal Steroids 2021  - Dr PadmeshUpdate on Antenatal Steroids 2021  - Dr Padmesh
Update on Antenatal Steroids 2021 - Dr PadmeshDr Padmesh Vadakepat
 
Inhaled Nitric Oxide (iNO) in Newborns - Dr Padmesh - Neonatology
Inhaled Nitric Oxide (iNO) in Newborns - Dr Padmesh - NeonatologyInhaled Nitric Oxide (iNO) in Newborns - Dr Padmesh - Neonatology
Inhaled Nitric Oxide (iNO) in Newborns - Dr Padmesh - NeonatologyDr Padmesh Vadakepat
 
Approach to Ano Rectal Malformations - Dr Padmesh - Neonatology
Approach to Ano Rectal Malformations - Dr Padmesh - NeonatologyApproach to Ano Rectal Malformations - Dr Padmesh - Neonatology
Approach to Ano Rectal Malformations - Dr Padmesh - NeonatologyDr Padmesh Vadakepat
 
Blood Group Selection in Newborn Transfusion - Dr Padmesh - Neonatology
Blood Group Selection in Newborn Transfusion  - Dr Padmesh - NeonatologyBlood Group Selection in Newborn Transfusion  - Dr Padmesh - Neonatology
Blood Group Selection in Newborn Transfusion - Dr Padmesh - NeonatologyDr Padmesh Vadakepat
 
Vaccination in Preterms by - Dr Padmesh - Neonatology
Vaccination in Preterms by  - Dr Padmesh - NeonatologyVaccination in Preterms by  - Dr Padmesh - Neonatology
Vaccination in Preterms by - Dr Padmesh - NeonatologyDr Padmesh Vadakepat
 
European Consensus Statement on RDS 2019
European Consensus Statement on RDS 2019European Consensus Statement on RDS 2019
European Consensus Statement on RDS 2019Dr Padmesh Vadakepat
 
Blood Brain Barrier by Dr Padmesh V
Blood Brain Barrier by Dr Padmesh VBlood Brain Barrier by Dr Padmesh V
Blood Brain Barrier by Dr Padmesh VDr Padmesh Vadakepat
 
Humidication in NICU - Dr Padmesh - Neonatology
Humidication in NICU - Dr Padmesh - NeonatologyHumidication in NICU - Dr Padmesh - Neonatology
Humidication in NICU - Dr Padmesh - NeonatologyDr Padmesh Vadakepat
 
Subgaleal Hemorrhage - Dr Padmesh - Neonatology
Subgaleal Hemorrhage - Dr Padmesh - NeonatologySubgaleal Hemorrhage - Dr Padmesh - Neonatology
Subgaleal Hemorrhage - Dr Padmesh - NeonatologyDr Padmesh Vadakepat
 
Touch and Massage Therapy in Newborn - Dr Padmesh V
Touch and Massage Therapy in Newborn - Dr Padmesh VTouch and Massage Therapy in Newborn - Dr Padmesh V
Touch and Massage Therapy in Newborn - Dr Padmesh VDr Padmesh Vadakepat
 
Perinatal infections- Diagnosis & Management - Dr Padmesh - Neonatology
Perinatal infections- Diagnosis & Management  - Dr Padmesh - NeonatologyPerinatal infections- Diagnosis & Management  - Dr Padmesh - Neonatology
Perinatal infections- Diagnosis & Management - Dr Padmesh - NeonatologyDr Padmesh Vadakepat
 
Shock & Inotropes in Neonates - Dr Padmesh - Neonatology
Shock & Inotropes in Neonates  - Dr Padmesh - NeonatologyShock & Inotropes in Neonates  - Dr Padmesh - Neonatology
Shock & Inotropes in Neonates - Dr Padmesh - NeonatologyDr Padmesh Vadakepat
 
Assessment of Fetal Well being - Dr Padmesh - Neonatology
Assessment of Fetal Well being - Dr Padmesh - NeonatologyAssessment of Fetal Well being - Dr Padmesh - Neonatology
Assessment of Fetal Well being - Dr Padmesh - NeonatologyDr Padmesh Vadakepat
 
Ballard score.. - Dr Padmesh - Neonatology
Ballard score..  - Dr Padmesh - NeonatologyBallard score..  - Dr Padmesh - Neonatology
Ballard score.. - Dr Padmesh - NeonatologyDr Padmesh Vadakepat
 
European Consensus Guidelines- RDS in Preterm Newborns
European Consensus Guidelines- RDS in Preterm NewbornsEuropean Consensus Guidelines- RDS in Preterm Newborns
European Consensus Guidelines- RDS in Preterm NewbornsDr Padmesh Vadakepat
 
Pulmonary Abscess in Children .. Dr Padmesh
Pulmonary Abscess in Children .. Dr PadmeshPulmonary Abscess in Children .. Dr Padmesh
Pulmonary Abscess in Children .. Dr PadmeshDr Padmesh Vadakepat
 

Más de Dr Padmesh Vadakepat (20)

Neonatal Nursing of Extremely Premature Neonates - Dr Padmesh
Neonatal Nursing of Extremely Premature Neonates - Dr PadmeshNeonatal Nursing of Extremely Premature Neonates - Dr Padmesh
Neonatal Nursing of Extremely Premature Neonates - Dr Padmesh
 
Update on Antenatal Steroids 2021 - Dr Padmesh
Update on Antenatal Steroids 2021  - Dr PadmeshUpdate on Antenatal Steroids 2021  - Dr Padmesh
Update on Antenatal Steroids 2021 - Dr Padmesh
 
Inhaled Nitric Oxide (iNO) in Newborns - Dr Padmesh - Neonatology
Inhaled Nitric Oxide (iNO) in Newborns - Dr Padmesh - NeonatologyInhaled Nitric Oxide (iNO) in Newborns - Dr Padmesh - Neonatology
Inhaled Nitric Oxide (iNO) in Newborns - Dr Padmesh - Neonatology
 
Approach to Ano Rectal Malformations - Dr Padmesh - Neonatology
Approach to Ano Rectal Malformations - Dr Padmesh - NeonatologyApproach to Ano Rectal Malformations - Dr Padmesh - Neonatology
Approach to Ano Rectal Malformations - Dr Padmesh - Neonatology
 
ROP - Dr Padmesh - Neonatology
ROP  - Dr Padmesh - NeonatologyROP  - Dr Padmesh - Neonatology
ROP - Dr Padmesh - Neonatology
 
Blood Group Selection in Newborn Transfusion - Dr Padmesh - Neonatology
Blood Group Selection in Newborn Transfusion  - Dr Padmesh - NeonatologyBlood Group Selection in Newborn Transfusion  - Dr Padmesh - Neonatology
Blood Group Selection in Newborn Transfusion - Dr Padmesh - Neonatology
 
Vaccination in Preterms by - Dr Padmesh - Neonatology
Vaccination in Preterms by  - Dr Padmesh - NeonatologyVaccination in Preterms by  - Dr Padmesh - Neonatology
Vaccination in Preterms by - Dr Padmesh - Neonatology
 
European Consensus Statement on RDS 2019
European Consensus Statement on RDS 2019European Consensus Statement on RDS 2019
European Consensus Statement on RDS 2019
 
Blood Brain Barrier by Dr Padmesh V
Blood Brain Barrier by Dr Padmesh VBlood Brain Barrier by Dr Padmesh V
Blood Brain Barrier by Dr Padmesh V
 
Humidication in NICU - Dr Padmesh - Neonatology
Humidication in NICU - Dr Padmesh - NeonatologyHumidication in NICU - Dr Padmesh - Neonatology
Humidication in NICU - Dr Padmesh - Neonatology
 
Subgaleal Hemorrhage - Dr Padmesh - Neonatology
Subgaleal Hemorrhage - Dr Padmesh - NeonatologySubgaleal Hemorrhage - Dr Padmesh - Neonatology
Subgaleal Hemorrhage - Dr Padmesh - Neonatology
 
Touch and Massage Therapy in Newborn - Dr Padmesh V
Touch and Massage Therapy in Newborn - Dr Padmesh VTouch and Massage Therapy in Newborn - Dr Padmesh V
Touch and Massage Therapy in Newborn - Dr Padmesh V
 
Perinatal infections- Diagnosis & Management - Dr Padmesh - Neonatology
Perinatal infections- Diagnosis & Management  - Dr Padmesh - NeonatologyPerinatal infections- Diagnosis & Management  - Dr Padmesh - Neonatology
Perinatal infections- Diagnosis & Management - Dr Padmesh - Neonatology
 
Shock & Inotropes in Neonates - Dr Padmesh - Neonatology
Shock & Inotropes in Neonates  - Dr Padmesh - NeonatologyShock & Inotropes in Neonates  - Dr Padmesh - Neonatology
Shock & Inotropes in Neonates - Dr Padmesh - Neonatology
 
ABC of ABG - Dr Padmesh
ABC of ABG - Dr PadmeshABC of ABG - Dr Padmesh
ABC of ABG - Dr Padmesh
 
Assessment of Fetal Well being - Dr Padmesh - Neonatology
Assessment of Fetal Well being - Dr Padmesh - NeonatologyAssessment of Fetal Well being - Dr Padmesh - Neonatology
Assessment of Fetal Well being - Dr Padmesh - Neonatology
 
Ballard score.. - Dr Padmesh - Neonatology
Ballard score..  - Dr Padmesh - NeonatologyBallard score..  - Dr Padmesh - Neonatology
Ballard score.. - Dr Padmesh - Neonatology
 
European Consensus Guidelines- RDS in Preterm Newborns
European Consensus Guidelines- RDS in Preterm NewbornsEuropean Consensus Guidelines- RDS in Preterm Newborns
European Consensus Guidelines- RDS in Preterm Newborns
 
Say NO to drugs .. Dr.Padmesh
Say NO to drugs .. Dr.PadmeshSay NO to drugs .. Dr.Padmesh
Say NO to drugs .. Dr.Padmesh
 
Pulmonary Abscess in Children .. Dr Padmesh
Pulmonary Abscess in Children .. Dr PadmeshPulmonary Abscess in Children .. Dr Padmesh
Pulmonary Abscess in Children .. Dr Padmesh
 

Último

call girls in Connaught Place DELHI 🔝 >༒9540349809 🔝 genuine Escort Service ...
call girls in Connaught Place  DELHI 🔝 >༒9540349809 🔝 genuine Escort Service ...call girls in Connaught Place  DELHI 🔝 >༒9540349809 🔝 genuine Escort Service ...
call girls in Connaught Place DELHI 🔝 >༒9540349809 🔝 genuine Escort Service ...saminamagar
 
Presentation on Parasympathetic Nervous System
Presentation on Parasympathetic Nervous SystemPresentation on Parasympathetic Nervous System
Presentation on Parasympathetic Nervous SystemPrerana Jadhav
 
SWD (Short wave diathermy)- Physiotherapy.ppt
SWD (Short wave diathermy)- Physiotherapy.pptSWD (Short wave diathermy)- Physiotherapy.ppt
SWD (Short wave diathermy)- Physiotherapy.pptMumux Mirani
 
Lippincott Microcards_ Microbiology Flash Cards-LWW (2015).pdf
Lippincott Microcards_ Microbiology Flash Cards-LWW (2015).pdfLippincott Microcards_ Microbiology Flash Cards-LWW (2015).pdf
Lippincott Microcards_ Microbiology Flash Cards-LWW (2015).pdfSreeja Cherukuru
 
Glomerular Filtration and determinants of glomerular filtration .pptx
Glomerular Filtration and  determinants of glomerular filtration .pptxGlomerular Filtration and  determinants of glomerular filtration .pptx
Glomerular Filtration and determinants of glomerular filtration .pptxDr.Nusrat Tariq
 
Culture and Health Disorders Social change.pptx
Culture and Health Disorders Social change.pptxCulture and Health Disorders Social change.pptx
Culture and Health Disorders Social change.pptxDr. Dheeraj Kumar
 
COVID-19 (NOVEL CORONA VIRUS DISEASE PANDEMIC ).pptx
COVID-19  (NOVEL CORONA  VIRUS DISEASE PANDEMIC ).pptxCOVID-19  (NOVEL CORONA  VIRUS DISEASE PANDEMIC ).pptx
COVID-19 (NOVEL CORONA VIRUS DISEASE PANDEMIC ).pptxBibekananda shah
 
Case Report Peripartum Cardiomyopathy.pptx
Case Report Peripartum Cardiomyopathy.pptxCase Report Peripartum Cardiomyopathy.pptx
Case Report Peripartum Cardiomyopathy.pptxNiranjan Chavan
 
Biomechanics- Shoulder Joint!!!!!!!!!!!!
Biomechanics- Shoulder Joint!!!!!!!!!!!!Biomechanics- Shoulder Joint!!!!!!!!!!!!
Biomechanics- Shoulder Joint!!!!!!!!!!!!ibtesaam huma
 
call girls in paharganj DELHI 🔝 >༒9540349809 🔝 genuine Escort Service 🔝✔️✔️
call girls in paharganj DELHI 🔝 >༒9540349809 🔝 genuine Escort Service 🔝✔️✔️call girls in paharganj DELHI 🔝 >༒9540349809 🔝 genuine Escort Service 🔝✔️✔️
call girls in paharganj DELHI 🔝 >༒9540349809 🔝 genuine Escort Service 🔝✔️✔️saminamagar
 
Wessex Health Partners Wessex Integrated Care, Population Health, Research & ...
Wessex Health Partners Wessex Integrated Care, Population Health, Research & ...Wessex Health Partners Wessex Integrated Care, Population Health, Research & ...
Wessex Health Partners Wessex Integrated Care, Population Health, Research & ...Wessex Health Partners
 
PNEUMOTHORAX AND ITS MANAGEMENTS.pdf
PNEUMOTHORAX   AND  ITS  MANAGEMENTS.pdfPNEUMOTHORAX   AND  ITS  MANAGEMENTS.pdf
PNEUMOTHORAX AND ITS MANAGEMENTS.pdfDolisha Warbi
 
Measurement of Radiation and Dosimetric Procedure.pptx
Measurement of Radiation and Dosimetric Procedure.pptxMeasurement of Radiation and Dosimetric Procedure.pptx
Measurement of Radiation and Dosimetric Procedure.pptxDr. Dheeraj Kumar
 
Big Data Analysis Suggests COVID Vaccination Increases Excess Mortality Of ...
Big Data Analysis Suggests COVID  Vaccination Increases Excess Mortality Of  ...Big Data Analysis Suggests COVID  Vaccination Increases Excess Mortality Of  ...
Big Data Analysis Suggests COVID Vaccination Increases Excess Mortality Of ...sdateam0
 
VarSeq 2.6.0: Advancing Pharmacogenomics and Genomic Analysis
VarSeq 2.6.0: Advancing Pharmacogenomics and Genomic AnalysisVarSeq 2.6.0: Advancing Pharmacogenomics and Genomic Analysis
VarSeq 2.6.0: Advancing Pharmacogenomics and Genomic AnalysisGolden Helix
 
Basic principles involved in the traditional systems of medicine PDF.pdf
Basic principles involved in the traditional systems of medicine PDF.pdfBasic principles involved in the traditional systems of medicine PDF.pdf
Basic principles involved in the traditional systems of medicine PDF.pdfDivya Kanojiya
 
Let's Talk About It: To Disclose or Not to Disclose?
Let's Talk About It: To Disclose or Not to Disclose?Let's Talk About It: To Disclose or Not to Disclose?
Let's Talk About It: To Disclose or Not to Disclose?bkling
 
Presentació "Real-Life VR Integration for Mild Cognitive Impairment Rehabilit...
Presentació "Real-Life VR Integration for Mild Cognitive Impairment Rehabilit...Presentació "Real-Life VR Integration for Mild Cognitive Impairment Rehabilit...
Presentació "Real-Life VR Integration for Mild Cognitive Impairment Rehabilit...Badalona Serveis Assistencials
 
PERFECT BUT PAINFUL TKR -ROLE OF SYNOVECTOMY.pptx
PERFECT BUT PAINFUL TKR -ROLE OF SYNOVECTOMY.pptxPERFECT BUT PAINFUL TKR -ROLE OF SYNOVECTOMY.pptx
PERFECT BUT PAINFUL TKR -ROLE OF SYNOVECTOMY.pptxdrashraf369
 
Pharmaceutical Marketting: Unit-5, Pricing
Pharmaceutical Marketting: Unit-5, PricingPharmaceutical Marketting: Unit-5, Pricing
Pharmaceutical Marketting: Unit-5, PricingArunagarwal328757
 

Último (20)

call girls in Connaught Place DELHI 🔝 >༒9540349809 🔝 genuine Escort Service ...
call girls in Connaught Place  DELHI 🔝 >༒9540349809 🔝 genuine Escort Service ...call girls in Connaught Place  DELHI 🔝 >༒9540349809 🔝 genuine Escort Service ...
call girls in Connaught Place DELHI 🔝 >༒9540349809 🔝 genuine Escort Service ...
 
Presentation on Parasympathetic Nervous System
Presentation on Parasympathetic Nervous SystemPresentation on Parasympathetic Nervous System
Presentation on Parasympathetic Nervous System
 
SWD (Short wave diathermy)- Physiotherapy.ppt
SWD (Short wave diathermy)- Physiotherapy.pptSWD (Short wave diathermy)- Physiotherapy.ppt
SWD (Short wave diathermy)- Physiotherapy.ppt
 
Lippincott Microcards_ Microbiology Flash Cards-LWW (2015).pdf
Lippincott Microcards_ Microbiology Flash Cards-LWW (2015).pdfLippincott Microcards_ Microbiology Flash Cards-LWW (2015).pdf
Lippincott Microcards_ Microbiology Flash Cards-LWW (2015).pdf
 
Glomerular Filtration and determinants of glomerular filtration .pptx
Glomerular Filtration and  determinants of glomerular filtration .pptxGlomerular Filtration and  determinants of glomerular filtration .pptx
Glomerular Filtration and determinants of glomerular filtration .pptx
 
Culture and Health Disorders Social change.pptx
Culture and Health Disorders Social change.pptxCulture and Health Disorders Social change.pptx
Culture and Health Disorders Social change.pptx
 
COVID-19 (NOVEL CORONA VIRUS DISEASE PANDEMIC ).pptx
COVID-19  (NOVEL CORONA  VIRUS DISEASE PANDEMIC ).pptxCOVID-19  (NOVEL CORONA  VIRUS DISEASE PANDEMIC ).pptx
COVID-19 (NOVEL CORONA VIRUS DISEASE PANDEMIC ).pptx
 
Case Report Peripartum Cardiomyopathy.pptx
Case Report Peripartum Cardiomyopathy.pptxCase Report Peripartum Cardiomyopathy.pptx
Case Report Peripartum Cardiomyopathy.pptx
 
Biomechanics- Shoulder Joint!!!!!!!!!!!!
Biomechanics- Shoulder Joint!!!!!!!!!!!!Biomechanics- Shoulder Joint!!!!!!!!!!!!
Biomechanics- Shoulder Joint!!!!!!!!!!!!
 
call girls in paharganj DELHI 🔝 >༒9540349809 🔝 genuine Escort Service 🔝✔️✔️
call girls in paharganj DELHI 🔝 >༒9540349809 🔝 genuine Escort Service 🔝✔️✔️call girls in paharganj DELHI 🔝 >༒9540349809 🔝 genuine Escort Service 🔝✔️✔️
call girls in paharganj DELHI 🔝 >༒9540349809 🔝 genuine Escort Service 🔝✔️✔️
 
Wessex Health Partners Wessex Integrated Care, Population Health, Research & ...
Wessex Health Partners Wessex Integrated Care, Population Health, Research & ...Wessex Health Partners Wessex Integrated Care, Population Health, Research & ...
Wessex Health Partners Wessex Integrated Care, Population Health, Research & ...
 
PNEUMOTHORAX AND ITS MANAGEMENTS.pdf
PNEUMOTHORAX   AND  ITS  MANAGEMENTS.pdfPNEUMOTHORAX   AND  ITS  MANAGEMENTS.pdf
PNEUMOTHORAX AND ITS MANAGEMENTS.pdf
 
Measurement of Radiation and Dosimetric Procedure.pptx
Measurement of Radiation and Dosimetric Procedure.pptxMeasurement of Radiation and Dosimetric Procedure.pptx
Measurement of Radiation and Dosimetric Procedure.pptx
 
Big Data Analysis Suggests COVID Vaccination Increases Excess Mortality Of ...
Big Data Analysis Suggests COVID  Vaccination Increases Excess Mortality Of  ...Big Data Analysis Suggests COVID  Vaccination Increases Excess Mortality Of  ...
Big Data Analysis Suggests COVID Vaccination Increases Excess Mortality Of ...
 
VarSeq 2.6.0: Advancing Pharmacogenomics and Genomic Analysis
VarSeq 2.6.0: Advancing Pharmacogenomics and Genomic AnalysisVarSeq 2.6.0: Advancing Pharmacogenomics and Genomic Analysis
VarSeq 2.6.0: Advancing Pharmacogenomics and Genomic Analysis
 
Basic principles involved in the traditional systems of medicine PDF.pdf
Basic principles involved in the traditional systems of medicine PDF.pdfBasic principles involved in the traditional systems of medicine PDF.pdf
Basic principles involved in the traditional systems of medicine PDF.pdf
 
Let's Talk About It: To Disclose or Not to Disclose?
Let's Talk About It: To Disclose or Not to Disclose?Let's Talk About It: To Disclose or Not to Disclose?
Let's Talk About It: To Disclose or Not to Disclose?
 
Presentació "Real-Life VR Integration for Mild Cognitive Impairment Rehabilit...
Presentació "Real-Life VR Integration for Mild Cognitive Impairment Rehabilit...Presentació "Real-Life VR Integration for Mild Cognitive Impairment Rehabilit...
Presentació "Real-Life VR Integration for Mild Cognitive Impairment Rehabilit...
 
PERFECT BUT PAINFUL TKR -ROLE OF SYNOVECTOMY.pptx
PERFECT BUT PAINFUL TKR -ROLE OF SYNOVECTOMY.pptxPERFECT BUT PAINFUL TKR -ROLE OF SYNOVECTOMY.pptx
PERFECT BUT PAINFUL TKR -ROLE OF SYNOVECTOMY.pptx
 
Pharmaceutical Marketting: Unit-5, Pricing
Pharmaceutical Marketting: Unit-5, PricingPharmaceutical Marketting: Unit-5, Pricing
Pharmaceutical Marketting: Unit-5, Pricing
 

OSCE Pediatrics KKCTH

  • 1. OSCE–kkcth Question No : 1 List 4 findings on the x- ray a) What is the radiological diagnosis?
  • 2. b) List 4 causes for the finding in the left lung c) List 4 causes for the finding in the left lung Answer for Question No : 1 a) Mediastinal shift (L) Pneumothorax (L) Atlectasis (L) lung Bronchopnemonic changes (R) b) Pneumothorax Bronchopneumonic changes (R) (Non – homogenous opacities c) 1) Pneumonia 2) Asthma 3) Foreign body in lung 4) Trauma
  • 3. 5) Cystic fibrosis 6) Tracheotomy 7) Subclavian line present 8) Thorococentesis 9) Transbronchial biopsy 10) Lymphomas & other malignancies 11) Ehlers Danlos syndrome 12) Marfan syndrome 13) Gangrene 14) Lung abscess Question No : 2 LINEZOLID 1) What group of drug is this? 2) Mode of action 3) Name 2 organisms for which this is a specific drug? 4) Dosage
  • 4. 5) Mention important hematological side effect? Answer for Question No : 2 1) Oxazolidinone 2) Translation initiation by blocking, formation of protein synthesis initiation complex by binding to 50 S ribosomal RNA.
  • 5. 3) MRSA vancomycin resistant Enterococci, coagulase negative staphylococci, penicillin resistant pneumococci 4) 10mg / kg / dose Q8 –12 hrs 5) Thrombocytopenia Question No : 3 QUINUPRISTIN / DALFOPRISTIN 1) What group of drug is this? 2) What is the mode of action? 3) Against which of the following organisms this is in effective?
  • 6. Mycoplasma, chlamydia, staph. aureus enterococcus faecalis. Answer for Question No: 3 1) Streptogramins Strepto gramins 2) Synergistic action on bacterial ribosomal subunit to protein synthesis 3) Enterococcus faecalis.
  • 7. Question No : 4 pH - 7.38 Pco2 - 38 BE -3 Hco3 - 21 Po2 - 98 1) Interpret the ABG? 2) List the indicators of compensation in ABG in following conditions.
  • 8. a) Metabolic Acidosis b) Metabolic Alkalosis c) Respiratory Acidosis – Acute/ Chronic d) Respiratory Alkalosis – Acute/ Chronic Answer for Question No : 4 1) Normal ABG  Met. Acidosis PCo2 = 15 x (HCo3) +8+/-2  Metabolic Alkalosis - PaCo2 increases by 7mm of Hg for each 10mq increases in the (HCo3 -)  Respiratory Acidosis Acute – (HCo3) increases by 1 for each 10mm increased in Pco2. Chronic – (Hco3) increases by 3.5 for each
  • 9. 10mm increase in PCo2.  Respiratory Alkalosis Acute – (HCo3) fall by 2 for each 10mm Hg decrease in PCo2. Chronic – (HCo3) decreased by 4 for each 10mm of decrease in PCo2. Question No : 5 1) You are asked to counsel a mother who’s 9 month of infant has AWD regarding ORT Check list: i) Introduces himself ii) Explains that the main treatment is ORT and explains the need for rehydration. iii) Explains correctly the preparation of ORT whole packet in 1 liter of water. iv) Advises feeding by spoon discourages bottle feeding. 2) Mother asks what to do if the baby vomits v) Stop ORT for 5 – 10minutes and restart feed, give slowly spoonful every 2 – 3 minutes vi) Advise giving small aliquots of 5 – 10ml each time.
  • 10. vii) Explains the danger signs of dehydration and explains when she should seek medical attention Does not become better in 3 days or develops danger Signs (Seizure / unconscious / rapid breathing etc). viii) Encourage continuance of breast feeds / normal feeds / home available feeds. ix) Checks, whether the mother has understood or not. x) Ask the mother whether there are any doubts. Answer for Question No : 5 1) You are asked to counsel a mother who’s 9 month of infant has AWD regarding ORT Check list: a. Introduces himself b. Explains that the main treatment is ORT and explains the need for rehydration. c. Explains correctly the preparation of ORT whole packet in 1 liter of water. d. Advises feeding by spoon discourages bottle feeding. 2) Mother asks what to do if the baby vomits v) Stop ORT for 5 – 10minutes and restart feed, give slowly spoonful every 2 – 3 minutes vi) Advise giving small aliquots of 5 – 10ml each time.
  • 11. vii) Explains the danger signs of dehydration and explains when she should seek medical attention Does not become better in 3 days or develops danger Signs (Seizure / unconscious / rapid breathing etc). viii) Encourage continuance of breast feeds / normal feeds / home available feeds. ix) Checks, whether the mother has understood or not. x) Ask the mother whether there are any doubts. Question No : 6 6 yrs old boy is brought for bed-wetting. His frequency in day time is normal. He is dry in the day. He is never been dry in the night. 1) Define this problem? What type is it?
  • 12. 2) Is it complicated or uncomplicated? Enumerate 4 differences between complicated and uncomplicated. 3) Name 3 drugs and dosage for the pharmacological therapy of this condition. Name 3 non – pharmacological measures for the management of this condition. Answer for Question No : 6 1) Nocturnal Enuresis. Primary Nocturnal Enuresis 2) Uncomplicated Uncomplicated Complicated Onset Primary Secondary Daytime symptoms absent + Stream Normal Abnormal
  • 13. Physical Normal Abnormal Urine analysis Normal Abnormal 3) Drug “ODI!!!” Dose DDAVP 10 – 40 mcg/day Nasal spray Oxybutinin 10 - 20 mg/day PO Imipramine 0.9 – 1.5 mg/kg/day Non-pharmacological Behavioral modification, Bladder exercises, alarm device. Question No : 7 List 3 abnormalities in this ECG 1) What is the ECG Diagnosis? 2) List 4 causes for the same
  • 14. 3) Drug of choice 4) Mention 1 complication Answer for Question No : 7 1) Fibrillary waves Absence of P waves Irregular Ventricular response / rhythm 2) Atrial Fibrillation 3) Rheumatic Valvular disease
  • 15. Thyrotoxicosis Following cardiac surgery Pulmonary embolism Pericarditis WPW syndrome Mitral regurgitation 4) Digoxin 5) Stroke / Thromboembolism Question No: 8
  • 16. 1) Identify the organism? 2) Name the method and steps used for the preparation for the smear?
  • 17. Answer for the Question No: 8 Acid-fast bacilli Ziehl neelsen technique CSEWM Heat and dry. Fix the smear. Add strong carbol fushcin Heat approximately for 5mins. Do not boil. Decolorise the smear with 20% sulphuric acid Decolorise with ethanol Wash with water Counter stain with methylene blue
  • 18. Question No : 9 1) Describe the pedigree 2) What is the mode of inheritance? 3) Give 4 examples.
  • 19. Answer for Question No : 9 1) 3 generation pedigree chart showing  All daughters of the affected males have the disease  Sons of the affected males are normal  Affected females affect ½ of the males and ½ of the daughters 2) X- linked dominant inheritance 3) Hypophosphatemic rickets ( Vit.D resistant)  Incontinentia pigmenti XD RHIO  Oro facial digital syndrome  Rett syndrome
  • 20. Question No : 10 The following food substances, which contain Vit.A, need to be arranged based on Vitamin A content from high to low.  Papaya,  Guava  Amaranth  Drumstick leaves  Egg  Human milk  Carrot
  • 21. Answer for Question No : 10 VIT A : CADEPM  Carrot 1167  Amaranth 515  Drumstick leaves 300  Egg 140  Papaya 118  Human Milk 38  Guava 0
  • 22. Question No : 11 1) Vitamin A prophylaxis programme in India - Mention the dosage and schedule. 2) Daily requirement of Vitamin A. 3) Name two manifestation of hypervitaminosis A?
  • 23. Answer for Question No : 11 1) 5 doses 9 months – 3 yrs Oral retinol palmitate 1lakh 9 months (along with measles) 2lakh 1½ yrs 2lakh 2 yrs 2lakh 2 ½ yrs 2lakh 3 yrs 2) 400mg to 600mg of requirement Retinal / RE B –carotene I.U 0-1yr 350mcg 1200mcg 1166.67 1-6yrs 400mcg 1600mcg 1333.33 >7yrs 600mcg 8400mcg 2000 1mcg = 3.3 IU 1 IU of vitamin A = 0.3mcg at retinal 3) Nausea, vomiting, anorexia, sleep distress, irritability Skin desquamation Hepatomegaly Pseudo tumor cerebri (diplopia/ papilledema /cranial N.Palsy) Alopecia, seborrhea,
  • 24. cutaneous leisions craniotabes Tender bony swellings Fissures at corners and mouth Question No : 12 Match the following: 1) BCG - Toxoid & killed bacteria 2) OPV - Live attenuated bacteria 3) DPT - Bacterial sub unit 4) Hib - Viral Antigen 5) Hep B Vaccine – Live attenuated viral 6) Typhoid V I - Killed virus 7) Hep A Vaccine – Capsular polysaccharide 8) Acellular pertusis – Capsular polysaccharide
  • 25. Question No : 13 Answer the following:  What is the Diluent for BCG?  What is the Diluent for MMR?  How long can reconstituted BCG be used?  How long can reconstituted MMR be used?  Name 5 vaccines which should not be frozen  What does IAP recommend at 5 yrs - (DPT/DT)?
  • 26. Answer for Question No: 12 1) BCG - Live attenuated bacteria 2) OPV - Live attenuated viral 3) DPT - Toxoid & killed bacteria 4) Hib - Capsular poly saccharide 5) Hep B Vaccine - Viral Antigen 6) Typhoid V I - Capsular poly saccharide 7) Hep A Vaccine - Killed virus 8) Acellular pertusis - Bacterial sub unit
  • 27. Answer for Question No : 13  Diluent for BCG is Sterile NS  Diluent for MMR is Distilled water  Reconstituted BCG can be used for 3 hrs  Reconstituted MMR can be used for 1 hr.  DPT, Hepatitis A & B, Varicella, Hib, TT  IAP recommends DPT at 5 yrs.
  • 28. What are indications for Acellular pertusis – vaccine? Mention 4 indications for pneumococcal vaccine?
  • 29. Answer for Question No : 13 Indications are:  Persistent / inconsolable Cry 3 or more hrs in 48 hrs.  Temperature > 40º within 48 hrs  Collapse / shock with (HHE within 48 hrs)  Convulsions with or without fever within 72 hrs of immunization.  Encephalopathy within 7 days, behavioral problems. 3) Indications:  Prior to splenectomy,  HIV  CSF Rhinorrhea  Sickle cell  Asplenia
  • 30.  CRF  Chronic lung / heart disease Question No : 14 A 6 year old girl has been referred for evaluation of anemia. Answer the questions after seeing the peripheral smear? a) What is your diagnosis?
  • 31. b) What would be the confirmatory test to clinch your diagnosis ? c) What is the definitive treatment of this condition ? Answer for Question No : 14 1) Hereditary spherocytosis 2) Incubated osmotic fragility 3) Splenectomy
  • 32. Question No : 15 You are asked to resuscitate a newborn with the provided equipments. Please ask questions regarding status of infant – wherever necessary. 1) Check the following equipments before proceeding further  Bag mask valve  Laryngoscope 2) Get information about the infant from the observer before proceeding to resuscitate and at each step whenever necessary Answer for Question No : 15
  • 33. 1) Check list for observer. Bag mask valve…does he - attach reservoir? - check pop off valve? - check for leak? Laryngoscope – Checks bulb & handle Does candidate ask the following 5 questions?  Meconium staining of liquor or not?  Term or preterm?  Crying well – breathing well or not?  Pink or blue colour?  Good muscle tone? The Observer Should Say Baby Is Not Breathing Does he clear airway/provide warmth/ position dry infant? 1..5 marks and then ask status of baby Observer Says: Baby Still Not Breathing well Does he give PPV for 30 seconds? Correct position EC clamp technique Chest expansion and then ask status of baby Observer Says Hr- 50/Min, Blue Does he start chest compressions? Correct technique?
  • 34. Question No : 16 1) What is the diagnosis? 2) This infant is 8 months old, what is the most likely type? 3) What is the earliest sign of this disorder? 4) What is the first radiological change that occurs in response to specific therapy? 5) How could this have been prevented? 6) What are the non – specific urinary findings
  • 35. in this disorder? (at least 2) Answer for Question No: 16 1) Rickets 2) Vitamin D deficiency 3) Craniotabes 4) Appearance of provisional zone of calcification 5) Supplement of 400IU of vitamin D 6) Generalized aminoaciduria  Glycosuria  Phosphaturia  Elevated urinary citrate
  • 36.  Impaired renal acidification. Question No : 17 You have performed ICD on a child with empyema. How will you dispose the used items given below?  Scalpel blade, hypodermic needles, trochar, used ampoules  Cotton, gauze, linens, suture material, surgical mask, gloves  Pus, 3 way connector  Syringe, plastic covers of gloves and ICD bag cover
  • 37. Answer for Question No : 17  Blue / white transparent puncture proof container  Yellow bag  Red bag  Black plastic bag.
  • 38. Question No : 18 This child has fever with URI 1) What is the diagnosis? 2) What is the causative organism? 3) Name one serious hematological complication in this disease. 4) Name one orthopedic complication. 5) Name one cause for intrauterine fetal demise.
  • 39. 6) What treatment is recommended for severe hematological complications? Answer for Question No : 18 1) Erythema Infection or fifth disease 2) Parvovirus B 19 3) Transient aplastic crisis 4) Arthropathy 5) Non – immune fetal hydrops 6) IvIg
  • 40. Question No : 19 Calculate the mean, median, mode and mean deviation of the diastolic pressures given below. 83,75,81,79,71,95,75,77,84
  • 41. Answer for Question No : 19 Mean = 80, Median = 79, Mode = 75 and Mean deviation = 5.1 The average of the deviations from arithmetic mean MD = ∑ (x -xˉ)/n ∑ = summation x = item values xˉ = Mean x - xˉ = deviation from mean n = No. of items
  • 42. Question No :20 Take relevant history from this parent whose child is suspected to have urinary tract infection for the first time.
  • 43. Answer for Question No : 20 Introduces himself History of fever History of constipation History of urgency History of malodorous urine History of suprapubic pain History of loin pain Details of coevute toilet training Wiping from back to front History of incontinence History of threadworm infection Family history of renal disease / stones
  • 44. Family history of UTI / VUR Note of thanks Question No : 21 You are asked to perform rapid sequence intubation. Write the steps sequentially. Mention the names of drugs wherever necessary.
  • 45. Answer for Question No : 21  Brief history and assessment  Assemble equipment, medications, etc.  Preoxygenate patient  Premedicate with lidocaine atropine  Sedation and analgesia induced  Pretreat with nondepolarizing paralytic agent  Administer muscle relaxants  Sellick maneuver  Endotracheal intubation  Secure tube, verify position with roentgenogram  Begin mechanical ventilation Step – 5 : Sedatives: Thiopental Diazepam Ketamine Analgesics: Fentanyl Morphine Succinylcholine Vecuronium or Pancuronium or rocuronium
  • 46. Question No : 22 A 2 year child presents with the following  5 episodes of abscesses in 6 months  Photosensitivity  Light skin and silvery hair Peripheral smear shows large inclusions in all nucleated blood cells. 1) What is the diagnosis? 2) What is the cause for the lighten hair? 3) What is the mode of inheritance? 4) Name one life threatening hematological complications? 5) What is the neurological manifestation?
  • 47. 6) Which drug is indicated? Answer for Question No : 22 1) Chediak – Higashi syndrome 2) Melanosomes or melanocytes are oversized. Failure to properly disperse the giant melanosomes to keratinocytes and hair follicles. 3) Autosomal recessive. Mutated gene for CHS Chromosome 1q2-q44. 4) Accelerated phase of a lymphoma like syndrome characterized by pancyopenia. 5) Peripheral neuropathy and ataxia Motor Sensory 6) High – dose ascorbic acid 200mg / 24hrs for infants 2,000mg/24hrs for adults. (2g !)
  • 48. Question No : 23 Perform Hand Washing
  • 49. Answer for Question No : 23 a) Remove ornaments / watch etc. hand sleeves above elbows. b) Perform six steps of hand washing 1) Palm to Palm 2) (i) Right palm over left dorsum (ii) Left palm over right dorsum 3) Fingers interlace palm to palm 4) Back of fingers to opposing palms 5) (i) Rotational rubbing of right thumb (ii) Rotational rubbing of left thumb 6) (i) Rotational rubbing of left palm (ii) Rotational rubbing of right palm c) Perform in 2 minutes d) Air dry / dry with sterile towel / paper e) Discard towel in black cover.
  • 50. Question No : 24 1) A Lumbar puncture is performed and the CSF is xanthochromatic. What are the four possible causes? 2) CSF protein levels are 400mg/dl. What are the three possible causes for the same? 3) CSF Glucose in 200mg/dl and blood glucose is 112mg/dl. List five causes for the same. 4) CSF is also cloudy, what does it imply?
  • 51. Answer for Question No : 24 1) a) Hyperbilirubinemia b) Subarachnoid hemorrhage c) Markedly elevated CSF protein d) Carotenemia 2) a) TB Meningitis b) GBS c) Tumors of spinal cord / brain d) Degenerative disorders e) Vasculitis f) Multiple sclerosis 3) a) Bacterial meningitis b) TBM c) Fungal meningitis d) Aseptic meningitis f) Neoplasms of meninges 4) Elevated WBC or RBC count
  • 52. Question No : 25 An adolescent presents with history of ingestion insecticides and has clinical features of organophosphorus poisoning. 1) Mention 2 methods of decontamination needed. 2) What is the mode of action of 2 antidotes used? 3) What 2 laboratory parameters are used to confirm the diagnosis of organophosphorus poisoning?
  • 53. Answer for Question No : 25 1) Activated charcoal for gastric decontamination. Skin decontamination by removal of clothes stained with organophosphorus. 2) Atropine – blocks acetylcholine receptor. Reverses the muscarinic and CNS effects. Pralidoxime (PAM) – breaks the bond between the organophosphate and the enzyme, liberating the enzyme and degrading the organophosphate. 3) Red cell cholinesterase and pseudo cholinesterase levels.
  • 54. Question No : 26 Answer the following questions after seeing the X-ray. 1) What are the findings? 2) Name 5 aerobic organisms, which can cause this?
  • 55. Answer for Question No : 26 1) Air and fluid filled cyst. Lung abscess, pneumatocele 2) Streptococcus Staphylococcus aureus Escherichia coli Klebsiella Pseudomonas
  • 56. Question No : 27 Dobutamine 1) What is the mode of action? 2) 7kg child requires Dobutamine infusion. How do you prepare the infusion? What is the dose? 3) What is T ½ of the drug / peak action? 4) Mention 3 contraindications? 5) Mention at least 6 adverse effects?
  • 57. Answer for Question No : 27 1) Act on the β1 adrenergic receptors of the myocardium. It increases stroke volume – increased COP – causes peripheral vasodilatation – decreases the sympathetic vascular tone – decreases the after load and there by improving the myocardial function. 2) Dose 2.5 – 15mcg/kg/min. infusion rate of 6mg/kg in 100ml normal NS, 1ml/hr will give 1mcg/kg/min. 3) T ½ - 2min, peak action 10 – 20min. ( Dobu2min ! ) 4) IHSS, atrial fibrillation and atrial flutter, sulfite sensitivity, hypotension. 5) Increase myocardial O2 demand tachycardia, ectopic heartbeat, angina / palpitations / tachyarrythmias, tingling sensation, parasthesia and leg cramps, diarrhoea and abdominal cramps.
  • 58. Question No : 28 4 year old child is being evaluated for syncope. 1) Identify the ECG? 2) Mention 2 acquired causes for the above abnormality? 3) Drug of choice. 4) What should be taught to parents?
  • 59. Answer for Question No : 28 1) Prolonged QT interval - > 0.45secds. 2) Myocarditis / electrolyte abnormality like calcium, mitral valve prolapse and drug induced. 3) Beta adrenergic antagonist- β blockers 4) Parents should be taught cardiopulmonary resuscitation.
  • 60. Question No : 29 1) a) List the components of IMNCI? b) List the components of reproductive and child health programme? 2) Mention 4 highlights of the Indian adaptation of IMNCI? 3)Which vaccine project has been introduced as a part of pilot project in IMNCI?
  • 61. Answer for Question No : 29 1a) Family planning Child survival and safe motherhood Client approach to health care Prevention and management of RTI /STD/ AIDS b) Improvement in case management skills of health staff, through provision of locally adopted guidelines and activities to promote their care. Improvement in overall health system Improvement in family and community health care system. 2) Inclusion of 0-7 days age in the programme Incorporating national guidelines on malaria Anemia, Vit.A supplementation, and immunization schedules Training of the health personnel begins with sick young infants upto 2 months Proportion of training time devoted to sick young infant and sick child is almost equal 3) Hepatitis B vaccine Question No : 30
  • 62. 1) What is the diagnosis? 2) What is the confirmatory test? 3) What neurological complications can occur? 4) Mode of inheritance? 5) What antibiotic is prescribed for this as prophylaxis?
  • 63. Answer for Question No : 30 1) Sickle cell anemia 2) Hemoglobin electrophoresis or HPLC 3) Stroke 4) Autosomal recessive 5) Penicillin
  • 64.
  • 65. Question No : 31 These lesions are tender 1) What is the diagnosis? 2) What 2 common infections and drugs can trigger this? 3) What 2 non – infectious systemic disorders can trigger this?
  • 66. Answer for Question No : 31 1) Erythema Nodosum 2) TB, Streptococcus Sulfa, Phenytoin, Oral contraceptives 3) IBD, Spondylo arthropathy, Sarcoidosis
  • 67. Question No : 32 You are asked to provide prophylaxis for bacterial endocarditis for 2 children with the following clinical details. Child 1: 8 year old boy with rheumatic mitral regurgitation is to undergo dental extraction tomorrow. 1) What is the drug of choice? 2) Dosage and timing Child 2: 2 years old male with VSD is to undergo Cystocopy tomorrow. 1) What is the drug of choice? 2) Dosage and timing. Child 3 :3 year old who has undergone PDA ligation 2 years back is to undergo dental extraction. What is the appropriate advice? Child4: 2 year old with TOF is to undergo circumcision. What is the appropriate advice?
  • 68. Answer for Question No : 32 1) Oral amoxycillin 50mg/kg 1hr before surgery (or) Ampicillin IV/IM 50mg/kg ½ hr before surgery 2) IV Ampicillin 50mg/kg + gentamycin 1.5mg/kg 30mins before surgery followed 6hrs later by IV/oral Ampicillin/amoxycillin 25mg/kg 3) No prophylaxis needed 4) No need for anti microbial prophylaxis.
  • 69. Question No : 33 Pedigree chart: 1) Identify the Mode of inheritance? 2) Give 3 examples of clinical disorders? 3) What is the significance of the pedigree symbols used in this?
  • 70. Answer for Question No : 33 1) Autosomal dominant 2) Neurofibromatosis, Huntington’s chorea, Myotonic dystrophy. 3) - Normal Male - Normal female - Affected male - Affected female - Proband - Dead
  • 71. Question No : 34 Pneumococcal 7 valent conjugate vaccine 1) What is the protein conjugate? 2) Route of administration? 3) Youngest age for administration? 4) Dosing interval? 5) Primary immunization schedule for infants < 6 months of age? 6) Dose if started at 12 – 23 months of age? 7) Dose if started > 24 months to 9 yrs of age?
  • 72. Answer for Question No : 34 1) Diphtheria CRM 197 protein 2) IM 3) 6 weeks 4) 4 to 8 weeks 5) 3 doses < 1 year 1 dose 12 to 15 months 6) 2 doses 7) 1 dose
  • 73. Question No : 35 A study was carried out to assess the utility of IgM Elisa test in the diagnosis of Leptospirosis. Blood culture positive cases were considered the gold standard for diagnosis. A total of 100 cases were studied. Leptospira were grown in blood culture in 40 of these cases. IgM Elisa was positive in 70 out of 100 cases. Out of these 70 cases, Leptospira were cultured in 30. IgM Elisa was negative in 30 cases, out of this 30, Leptospira was grown in culture in 10 cases. Calculate the following for IgM Elisa as a diagnostic test for Leptospirosis. 1. Specificity 2. Sensitivity 3. Positive Predictive Value 4. Negative Predictive Value
  • 74. Answer for Question No : 35 Blood c/s Blood c/s + - IgM elisa + 30 (a) 40 (b) 70 IgM elisa - 10 (c) 20 (d) 30 40 60 100 1) Specificity: d x 100 20 x 100 = 33.3% d+b 20 + 40 2) Sensitivity: a x 100 30 x 100 = 75% a+c 30 + 10 3) Positive predictive value: a x 100 30 x 100 = 42.85% a+b 30 + 40 4) Negative Predictive value: d x 100 20 x 100 = 66.6% c+d 10 + 20 Question No: 36
  • 75. A 2 year old child presents with biphasic fever, severe arthragia and rash tourniquet test is negative. Platelet count is normal. Hb is 10.28gm. 1) What is the most probable diagnosis? a) Dengue hemorrhagic fever b) Measles c) Chikunguniya fever d) Roseola infantum 2) List 2 criteria for case definition of this probable case and4 criteria for confirmed case 3) What neurological complication can develop? 4) What family does this virus belong to? Answer for Question No : 36
  • 76. 1) Chikunguniya 2) • Features of suspect case-Fever with chills / arthralgia / rash / rheumatic manifestations. • Case definition – features of suspect case and positive serology in acute and convalesce phase. • Confirmed case – probable case with any of the following : a) 4 fold rise in antibodies in paired sera b) Positive IgM c) Virus isolation from serum d) Positive RT PCR in serum e) Positive RT PCR in serum 3) Meningo encephalitis 4) Toga viridiae Question No : 37
  • 77. A 10 year old female weighing 30kg; diagnosed case of IDDM on insulin therapy as follows: 10 (regular) Morning: 40 units 30 (lente) 7 (regular) Evening: 20 units 13 (lente) Her recent morning blood sugars are becoming high. (Blood sugar at 7.00 am 280 mg%) 1) What is Somogyi and Dawn phenomenon? 2) How will you differentiate these two in this case? 3) How will you treat in either case? Answer for Question No : 37
  • 78. 1) Somogyi phenomenon: Hyperglycemia begetting hypoglycemia due to counter regulating hormones in response to insulin induced hypoglycemia. Dawn phenomen: Hyperglycemia (early morning) without preceding hypoglycemia due to decreased availability of insulin and increased GH release. 2) Measure blood sugar at 3 am, 4 am, and 7 am. • If blood sugar > 80mg/dl in 1st two sample, and high in 3rd Diagnosis – Dawn phenomenon • If blood sugar <60mg/dl in first two sample and high in 3rd. Diagnosis – Somogyi phenomenon 3) Treatment- • Dawn phenomenon – increased evening dose of Lente insulin by 10 – 15% • Somogyi phenomenon – decreased evening dose of Lente insulin by 10-15%. Question No : 38
  • 79. Councell the mother of a child who is being discharged from your hospital following acute severe asthma. Answer for Question No : 38
  • 80. 1) Introduces himself 2) Clearly explains about asthma as hyperactive airway disease and not infective. 3) Explains that there is no curative treatment and treatment reduces the severity and complications. 4) Explains how to use MDI. 5) Explains preventive strategies at home. 6) Explains danger signs / warning signs of acute attack. 7) Tells the treatment at home and reach nearest hospital. Tells difference between Rescue and prophylactic inhalers. 8) Explains other alternatives, and ask for any doubts and clears it. 9) Need for regular follow up. 10) Note of thanks and availability.
  • 81. Question No : 39 1) A child has massive GI bleed. His clotting time is 12 mins. His PT test is 40 secs, control 14secs and PTT test is 60 secs, control is 30 His Hb is 11gm and Platelet count is 2.5lakhs. What blood product would you transfuse? And how much? 2) List 4 other indications for transfusion with this blood product? 3) List 4 clotting factors deficiencies, which will be corrected by this transfusion? 4) Name 2 parasitic disease transmitted by blood transfusion?
  • 82. Answer for Question No : 39 1) Fresh Frozen Plasma - 15ml/kg 2) a) Severe clotting factor deficiency and bleeding b) Severe clotting factor deficiency and invasive procedure. c) Emergency reversal of warfarin effects d) Dilutional coagulopathy and bleeding. e) Anticoagulant protein (AT – III, protein C, Protein S, FTTP) 3) II, V, X and XI 4) Malaria / Chagas disease
  • 83. Question No : 40 A 13 year old HIV positive boy is seen in OPD for abrasions injuries over left thigh following RTA 2 days back. He was vaccinated with TT at age of 10 yrs. 1) What immunization advice is appropriate? 2) Mention the dose and route? 3) What is the amount of Tetanus toxoid present in DPT, DT and TT? 4) What is the preservative in TT? 5) What is the role of Aluminum Phosphate in TT?
  • 84. Answer for Question No : 40 1) a) Clean the wound with soap and water. b) Administer TT & TIG c) Advice booster dose. 2) a) TT 0.5ml IM on one buttock b) TIG 500U IM on opposite buttock 3) 10 Lf of TT component. 4) Thiomerosol 0.01% w/v 5) TT is adsorbed on to aluminum compounds and increases potency, by reaching high titers. It also gives long lasting immunity.
  • 85. Question No : 41 An infant is being evaluated for ambiguous genitalia. You find clitoral hypertrophy and other signs of virilization. On investigations Sr. cortisol levels are low. ACTH & PRA are markedly elevated. ACTH skin stimulation test reveals markedly increased 17 – OH progesterone. Serum testosterone is also elevated. Child also has severe hyponatremia. 1) What is your diagnosis? 2) What is the mode of inheritance? 3) What is pre-natal diagnosis? 4) What advice will you give her for the next pregnancy? 5) What treatment you offer for this baby?
  • 86. Answer for Question No: 41 1) Congenital adrenal hyperplasia due to 21 hydroxylase deficiency 2) Autosomal recessive 3) 1sttrimesterCVS,2ndtrimester amniocentesis for DNA common mutations or polymorphic micro satellite markers if affected siblings samples are available for comparison. 4) Dexamethasone 20mg/kg pre pregnancy maternal wt in two – three divided doses. Prefer CVS. continue treatment if female child. 5) Hydrocortisone 10 to 20mg/m2/day tds. (Increase in stress situation) Mineralo corticoids 0.1 – 0.3mg/day BD Sodium supplements 1 – 3gm. Surgical correction.
  • 87. Question No : 42 Drug: Carbamzepine 1) Mention 4 clinical indications? 2) In which type of seizures it is avoided? 3) What is therapeutic drug levels in blood and recommended time to draw sample? 4) Mention 4 common drugs, which increases its toxicity? 5) What dose adjustment is needed in ARF? 6) What is the standard concentration in suspension? 7) Mention dosage of frequency of suspension? 8) What & how frequent lab monitoring is needed? 9) Mention 4 life threatening complications? 10) What is treatment of toxicity?
  • 88. Answer for Question No : 42 1) GTCS, partial seizures, trigeminal neuralgia, bipolar disorders. 2) Myoclonic seizures 3) 4 –12mg/L, 30min. before oral dose. 4) Erythromycin, INH, TCA, clozapine, itraconazole, cimetidine. 5) Mild, mod forms - dose adjustment is not necessary, severe (Creatnine clearance < 10; decrease dose by 75%) 6) 5ml = 100mg 7) 10-20mg/kg/day, increased 100mg/day at 1 week interval (BD/QD). 8) CBC, SGOT, SGPT every monthly for first 3-4 months and then as needed. 9) Hyper sensitivity reactions, aplastic anemia, pancytopenia, hepatic toxicity, thrombocytopenia. 10) Gastric lavage repeated dose of activated charcoal, hemoperfusion or hemodialysis.
  • 89. Question 1 ½ year old male with acute gastroenteritis develops anuria. His Hb is 8.9, Platelet count 90,000 & Pt – 24/20, PTT – 28/30. His PS is shown below. 1) Describe PS 2) Mention two MC D/D for it. 3) Which single test will help differentiate your D/B? 4) This baby’s renal functions de--erated after 2 units of FFP. What would be the possibility? 5) What are indications of steroids in this scenario? 6) Which other conditions here similar PS findings?
  • 90. Answer 1) Microangiopath--- hemolytic anemia with helmetcells, burn cells, he------- RBCs 2) HUS, RVT 3) Doppler USG ------- 4) HUS due to strep pneumonia 5) Seizures 6) Malignment HTN, SLE
  • 91. Question A 10 year old male with acute onset progressive lower limb weakness. On detailed CNS examination you find he is cons--- alert and normal cranial nerve examinations. No bowel / bladder involvement. His knee and ankele re--- are brisk and has grade II power on both lower limbs. You also notice abdominal reglesses below ----- is absent alibbus sensory system normal. 1) what is your progressive diagnosis? 2) Mention 4 conditions where you get hypotension and diminished referes in UMN lesion? 3) Mention 3 major points to differentiated extra modular from intramedially lesions? 4) Mention a congential conditions leading to non compressive myelopathy?
  • 92. Question No : 43 A 8 year old boy is brought because his mother feels he is short for his age. His height is 80cm. His father’s height is 160cm and mother’s height is 148cm. His US/LS ratio is 1. 4: 1 1) What type of short stature does this child have? 2) What is the mid parenteral height of this child? 3) Name 3 causes for the short stature in this child? 4) What is the normal US/LS ratio at this age? 5) Name 3 conditions in which there is advanced US/LS ratio?
  • 93. Answer for Question No : 43 1) Dysproportionate dwarfism 2) 160cm 3) Achandroplasia, cretinism, short limb dwarfism. 4) 1.1:1 5) Arachynodactyl, chandrodystrophy, spinal deformity and eunochodism
  • 94. Question No : 44 An infant with seizures is being investigated. The following are the lab reports. Serum Calcium: 6.6mg% Po4: 9mg% SAP: 500 units Mg: 3mg% 1) What is the probable diagnosis? 2) What will be the levels of PTH & 1, 25(OH2) D3? 3) The same infant is also noted to be dark and having mucocutaneous candidiasis. What is your diagnosis? 4) CT brain is carried out. What finding do you expect?
  • 95. Answer for Question No : 44 1) Hypoparathyroidism 2) Both are low 3) Type I polyendocrinopathiy (with Addison’s) 4) Basal ganglia calcification
  • 96. Question No : 45 A 16 week infant is examined for developmental assessment. Write what patterns of behaviour will you expect in this age?
  • 97. Answer for Question No : 45 Prone : Lifts head and chest, arms extended. Ventral suspension : head above plane of body Supine : TNR and reaches toward and misses objects. Sitting : No head lag on pulling, head steady, tipped forward, enjoys sitting with truncal support. Standing : when held erect, pushes with feet. Adaptive : Sees pellet, makes no move to it Social : Laughs out loud, excited at sight of food, may show displeasure if social contact is broken.
  • 98. Question No : 46 A child with meningoenclphalitis is comatose. His serum sodium is 116 and you are contemplating diagnosis of SIADH 1) Which of the following lab values will be present? a) Urine OSM <100 mosm/l b) Plasma volume normal or increased c) Urine Na 200mg/L d) Serum uric acid 10mg% 2) Name 2 drugs which increases the vasopressin levels? 3) The fluid intake should be restricted to 4) Which drug may be given? 5) 2 acute respiratory illnesses, which cause this?
  • 99. 6) Anticonvulsant, which decreases ADH production? Answer for Question No : 46 1) a) Urine OSM <100 mosm/l - Negative b) Plasma volume normal or increased - Positive c) Urine Na 200mEq/L - Positive d) Serum uric acid 10mg% - Negative 2) CBZ, Vincristine, TCA 3) 1000ml/ m2 /24hr 4) Demeclocycline 5) Broncholitis, pneumonia 6) Phenytoin
  • 100. Question No : 47 1) Identify this. 2) What is the route of entry of this organism? 3) Name 2 conditions that are high risk for this infection? 4) What hematological clue will occur? 5) Drug of choice? 6) 1 complication
  • 101. Answer for Question No : 47 1) Strongyloides stercoralis Larvae 2) Skin 3) HIV/ Immunosuppression, PEM, MR, Autoimmune disease 4) Eosinophilia 5) Ivermectin 200mcg/kg OD for 1 – 2 days 6) Hyperinfection syndrome
  • 102. Question No : 48 A child with stroke is noted to have Ectopia Lentis, arachinodactyly blue eyes and developmental delay. 1) What is the likely diagnosis? 2) What urine screening test will be positive? 3) Estimation of plasma amino acids will show ↑ cystine ↑ Methionine ↑ Homocystine 4) Which vitamin is indicated? 5) What other drug is needed?
  • 103. 6) Mode of inheritance? Answer for Question No : 48 1) Homocystinuria 2) Positive cyanide nitroprusside test 3) ↑ cystine - Negative ↑ Methionine - Positive ↑ Homocystine - Positive 4) B6 200 – 1000mg/24hr 5) Betaine 6) AR
  • 104. Question : 49 You are to meet a child with Thalassemia major 1) When do you start transfusion? 2) Optimum Hb 3) Intervals for transfusion 4) What is the risk in keeping Hb > 14? 5) When will HbF be low? 6) How do you reduce non-hemolytic febrile transfusion? 7) What is the level of serum ferritin to be maintained? 8) Indications for splenectomy
  • 105. 9) When to give vaccines before splenectomy? 10) Antenatal diagnosis How and When? Answerfor Question No : 49 1) < 7gm 2) 9 – 10gm 3) 2 – 4 weeks 4) Thrombosis 5) After repeated transfusions 6) Leukocyte filter, pheneramine with paracetamol 7) < 1000mg/L 8) > 220ml/kg/yr of PRBCs, hypersplenism, massive spleen with prominent abdominal discomfort. 9) Atleast 4 weeks
  • 106. 10) CVS- mutation, 12to 14 weeks (MTP < 20weeks) Question No : 50 Match the disease with the Urine Screening Test 1) Galactosemia Nitroprusside test 2) PKU CN PT 3) MPS DNPH 4) HCU Benedicts test 5) Organic aciduria spot test (Toludene blue test) 6) Cystinuria Fecl2
  • 107. Answer for Question No : 50 1) Galactosemia Benedicts test 2) PKU Fecl2 3) MPS MPS spot test (Toludene blue test) 4) HCU (Nitroprusside test) 5) Organic aciduria DNPH 6) Cystinuria CNPT
  • 108. Question : 51 DRUG: Vigabatrine 1) Mode of action? 2) Dosage 3) Important side effects 4) Mention important uses
  • 109. Answer for Question No : 51 1) Y aminobutyric acid, transaminase inhibitor 2) 30mg/kg/d, od /bd upto 100mg/kg/d 3) Visual field constriction, Optic atrophy, optic neuritis 4) Infantile spasms, Tuberous sclerosis adjuvant for poorly controlled seizures.
  • 110. Question No : 52 A 2 year old child is brought for Toe walking. 1) What is the commonest cause? 2) What is the Differential diagnosis? 3) Upto what age is it normal?
  • 111. Answer for Question No: 52 1) Normal children 2) Cerebral palsy, Duchenne muscular dystrophy, tethered cord, congenital tendo-achilles contracture, leg-length discrepancy, CDH and habitual 3) 3 year
  • 112. Question No : 53 1) What is premature thelarche? 2) Upto what age is it benign? 3) What is exaggerated or atypical thelarche? 4) What will be the level of FSH, LH, oestradiol in benign premature thelarche? 5) What will be the USG findings?
  • 113. Answer for Question No : 53 1) Isolated breast development 2) < 3 yrs 3) Associated with accelerated bone age due to systemic ostrogen effects 4) Low 5) Small Ovanian cysts
  • 114. Question No: 54 A child has chronic polyarthritis of 4 joints and is ANA positive. 2) What arthritis is it likely to be? 3) What complication should we anticipate? 4) In which type of JRA is HLA β27 positive ? 5) A child with JRA presents with fever, leuepenia and hepatosplenomegaly and lymphadenopathy. What is the diagnosis? 6) What drug is indicated in treatment of Q4?
  • 115. Answer for Question No: 54 1) JIA pauciarticular type I 2) Chronic uveitis 3) Pauciarticular type II 4) MAS 5) Cyclosporin
  • 116. Question No :55 Oxygen therapy 1) Below what blood O2 does WHO recommend O2 therapy? 2) What are the clinical indications for O2 therapy? 3) O2 concentration with reference to FiO2 Nasal prongs @1-2 lit/min, Nasopharyngeal catheter
  • 117. Answer for Question No : 55 1) <90% 2) Central cyanosis, unable to drink due to respiratory distress. In those with pneumonia, Broncholitis and asthma- severe lower chest in drawing, RR >70, Grunting, Head nodding 3) FiO2 30 – 35 %, 45 – 60 %
  • 118. Question No : 56 A child is brought with snakebite. 1) In which of the following is appropriate as first aid. a) Splint the limb b) Apply ice c) Apply tourniquet to occlude venous flow d) Clean the wound e) Transport to hospital 2) 2 specific indication for ASV 3) What is the dose of ASV to a 3 year old? 4) What is the diluent for ASV?
  • 119. Answer for Question No : 56 1) a) Splint the limb - b) Clean the wound – c) Transport to hospital – 2) Indications: a) Systemic signs of envenomation b) Local symptoms like severe necrosis, swelling of > half of the limb 3) Same as adult 4) Normal saline (2 to 3 volumes)
  • 120. Question No: 59 1) What are the 4 types of lesions? 2) What bacteria cause this? 3) Which drug can induce this? 4) Name 4 drugs used? 5) What Dietary advice will you give?
  • 121. Answer for Question No : 59 1) Open Comedones – blackhead Closed comedones – whitehead Papules, pustules and nodulocystic lesions 2) Propionibacterium acnes 3) Corticosteroid, androgens, INH, phenobarbital, Phenytoin, B12 and lithium 4) Benzoyl Peroxide, Tretinoin, Adapalene, Topical – Erythromycin and clindamycin Use for 4 to 8 weeks: Systemic therapy: Tetracycline, Doxy, Minocycline Isotretinoin (nodulocystic) (teratogenic) Intradermal triamcinolone 5) Normal
  • 122. Question No : 57 A child presents with muscle cramps. The serum Magnesium is 1mg/dl, K is 1.6mg/dl and Hco3 is 40mg/dl. There is no dehydration. 1) What electrolyte in urine will you estimate? 2) Why? 3) The level of the urinary electrolyte estimated is high – BP is normal. List 3 possible diagnosis? 4) There is no history of drug ingestion or failure to thrive hypertension. What is the diagnosis? 5) 3 drugs for treatment 6) What will be level of renin and aldosterone in serum? 7) What will be the urinary calcium level?
  • 123. Answer for Question No : 57 1) Calcium 2) To distinguish low and high urine calcium levels 3) Barter’s, giltelman’s and base administration 4) Giltelman syndrome 5) Na, mg and spiranolactone 6) Normal 7) Low
  • 124. Question No : 58 A child is admitted with TCA poisoning. 1) What are the 3 ‘C’s in manifestations? 2) What ECG findings do you anticipate? (3) 3) Which of the following is correct? a) Emesis is indicated b) Activated charcoal to be given c) Na Hco3 must d) PH should be 7.45 to 7.55 e) Lidocaine not be used for any time f) Quinidine & procainamide - to be used g) NaHco3 is used to prevent cardiac arrhythmias
  • 125. Answer for Question No : 58 1) Coma, convulsion, cardiac toxicity 2) Widening of QRS, Q-T prolongation,-, flat or inverted T , ST depression, RBB, CHB 3) A) No b) Yes c) Yes d) Yes e) No f) No g) Yes
  • 126. Question No : 61 1) What is the diagnosis? 2) Name 2 topical agents of use? 3) Duration and frequency? 4) 2 drugs for systemic therapy? 5) Commonest organisms?
  • 127. Answer for Question No : 61 1) Tinea corporis 2) Miconazole, ketoconazole, clotrimazole, econazole, terbinafine, niftifine 3) Bd, 2 to 4 week 4) Griseofulvin – several weeks Itraconazole – 1 – 2 week 5) T. Rubrum, T. Mentagrophytes
  • 128. Question No : 60 Write the calorie value of Rice – 1 cup Puri – 1 Upma – 1 cup Idli – 1 Dosa – 1 Kichidi - 1 cup Boiled egg - 1 Vada - 1 Pizza – 1 slice Oil – 1 tbsp Ice cream – ½ cup Peanuts – 50 nos Banana – 1 Cashew nuts – 10 Milk chocolate – 25gm
  • 129. Answer for Question No : 60 Rice – 1 cup - 170 Puri – 1 - 100 Upma – 1 cup - 270 Idlli – 1 - 75 Dosa – 1 - 125 Kichidi - 1 cup - 200 Boiled egg – 1 - 90 Vada – 1 - 70 Pizza – 1 slice - 200 Oil – 1 tbsp - 60 Ice cream – ½ cup - 200 Peanuts – 50 nos - 90 Banana – 1 - 90 Cashew nuts – 10 - 95 Milk chocolate – 25gm - 140
  • 130. Question No : 62 This girl has palpitation, diarrhea, and loss of weight 1) What is the diagnosis? 2) The thyroid swelling is not tender, not nodular. She has exophthalmos. What is the cause? 3) List 4 other causes? 4) Investigations 5) 2 drugs used
  • 131. Answer for Question No : 62 1) Hyperthyroidism 2) Graves Disease 3) Toxic adenoma, toxic multinodular , subacute thyroiditis, Lymphocytic thyroiditis, iodine induce, exogenous hormone, pituitary adenoma and ovarian tumor. 4) TSH is decreased, T4 increased. Thyroid uptake increased 5) β blockers, methimazole (propylthiouracil)
  • 132. Question No : 63 A spirometry is performed in an asthmatic child 1) What will be abnormalities in the following: FEV1 FEV1 / FVC Improvement in FEV1 Exercise challenge 2) What PEFR variation that is consistent with a diagnosis of asthma?
  • 133. Answer for Question No : 63 1) FEV1 – Low FEV1 / FVC ration < 0.8 Improvement in FEV1 with inhaled β2 agonist ≥12% Exercise challenge – worsening in FEV1 ≥ 15% 2) Morning to afternoon variation ≥ 20%
  • 134. Question No : 64 The following is the ABG is an infant PH - 7.2, Hco3 – 10, CO2 – 30 1) What is the diagnosis? 2) Why? 3) Give an example of a common clinical setting for this condition? 4) Write the other compensation in acid bone disorder. Which are appropriate?
  • 135. Answer for Question No : 64 1) Metabolic acidosis – respiratory acidosis 2) Expected Co2 = 1.5 X (10) + 8 ± 2 = 21 to 25 Co2 >25 3) Pneumonia with sepsis (lactic acidosis and respiratory acidosis) 4) Metabolic alkalosis Pco2 ↑ 7mm for 10meq/L of Hco3 Respiratory acidosis – Acute: Hco3 ↑ 1 for 10mm ↑ in Pco2 Chronic: Hco3 ↑ 3.5 for 10mm ↑ in Pco2 Respiratory alkalosis Acute: Hco3 ↓ 2 for 10mm ↓ in Pco2 Chronic: Hco3 ↓ 4 for 10mm↓ in Pco2
  • 136. Question No : 65 Serum Na - 136, Cl - 102 and Hco3 - 10 1) What is the anion gap? 2) What is the normal anion gap? 3) In which of the following is anion gap normal or increased? Diarrhea Lactic acidosis DKA ARF RTA Salicylate poisoning Urinary tract diversion IEM Septic shock Post hypocapnea
  • 137. Answer for Question No : 65 1) (136) – (102 + 10) = 24 2) 8 – 16 3) Diarrhea - Normal Lactic acidosis - increased DKA - increased ARF - increased RTA - Normal Salicylate - increased Urinary tract diversion - Normal IEM - increased Septic shock - increased Post hypocapnea - Normal
  • 138. Question No : 66 1) What is the mode of inheritance? 2) 2 examples 3) Characteristics of the inheritance 4) Plasma ammonia in this child is normal. What is the likely diagnosis? 5) There is no odor or skin lesion, but there is ketosis. What is the likely diagnosis?
  • 139. Answer for Question No : 66 1) XLD 2) VDRR (Hypophosphotemia), incontinentia pigmenti 3) Affected men all affected All normal All + of affected have 50% inheritance Rare XLD - milder disease Thrice as common as male 4) Organic acidemia Urea cycle – NH3 , anion gap – normal Amino acid defects or Galactosemia NH3 – normal, anion gap – normal
  • 140. 5) MMA, Propionic acidemia, Ketothiolase deficiency Odor + - MSUD / isovaleric Skin + - Multiple carboxylase deficiency Ketosis - Acyl CoA, 3Hydroxy 3HGA, HMG co - synthetase deficiency
  • 141. Question No : 67 1) What is the clinical classification of leprosy in India? 2) What are the differences in leprosy constitution? 3) What is the WHO recommended standard treatment regime for children aged 10 – 14 yrs?
  • 142. Answer for Question No : 67 1) Indeterminate Tuberculoid Paucibacillary 1 to 5 lesions Borderline in skin Lepromatous Multi bacillary > 5 lesions Pure neuritis 2) Case of leprosy – clinical sign + Bacilli in smear + Not completed treatment Paucibacillary – 1 to 5 lesions Multibacillary Adequate treatment Repeated treatment
  • 143. Newly diagnosed case Defaulter Relapsed case 3) Multi Bacillary (completed in 12 months) RMP 450mg once a month Clofazamine 150mg once a month (supervised) Clofazamine 50mgEod (self administered) Dapsone 5omg once a month (supervised) Dapsone 50mg daily dose ( domiciliary) Pauci Bacillary (completed in 6 months) Rifampicin 450mg once a month Dapsone 50mg OD daily (domiciliary)
  • 144. Question No : 68 Midday meal program 1) What are the principles? 2) Write a model menu?
  • 145. Answer for Question No : 68 1) Meal – a supplement, not a substitute to home diet Should supply 1/3 of total energy, ½ of protein Cost low Easily cookable in school Use locally available foods Change menu frequently 2) Cereals & and millets – 75gms Pulses - 30 Oil & fat - 8 Leafy vegetable –30 Non-leafy - 30
  • 146. Question No : 69 Answer the following questions with regard to cold chain equipment: 1) Which vaccines are stored in deep freezers? 2) Which vaccines are stored in ILR? 3) Which vaccines are kept in the basket of ILR? 4) What is the function of cold boxes? 5) What are Day carriers used for? 6) What are vaccine carriers used for? 7) Which vaccines should not be frozen?
  • 147. Answer for Question No : 69 1) Measles & OPV 2) All 3) TT, DPT, DT and diluents (not in the floor – may be frozen) 4) Transportation of vaccines 5) Carry small quantities of vaccines to a nearly session. 6) Carry small quantities of vaccines to out of reach session 7) DPT, DT, TT, Typhoid, BCG, HBV, diluent
  • 148. Question No : A 11 year old boy is brought with a penetrating crush injury with a compound fracture. His immunity and immunization status for tetanus is unknown. Which of the following action is correct with regard to tetanus prevention? a) Nothing is required b) Toxoid 1 dose c) Toxoid 1 dose + TIG d) Toxoid complete course + TIG His 8 year old sister has multiple clean abrasions. She has earlier received 3 doses of DPT in the first year and 1 booster at 1 ½ year and no other vaccines after that What prevention will you carry out?
  • 149. Answer for Question No : All wounds – surgical t------ < 6hr, clean, non penetrating with negligible tissue damage a) Nothing b) TT 1 c) TT 1 d) TT complete course All wounds – surgical t------ other wounds immunity category. a) Nothing b) TT 1 c) TT 1 + TIG d) TT complete course + TIG a) Complete course + Bon----- < 5 yrs b) Complete course + 5 to 10 yrs c) Complete course + > 10 yrs d) Immunity unknown + has not heel a complete course of toxoid.
  • 150. Question No : 70 The peak flow rates of 10 children of same age are as follows: 250, 260, 290, 200, 240, 240, 260, 270, 270, 290 1) What is the range? 2) What is the mean deviation and Mean? 3) What is the standard deviation?
  • 151. Answer for Question No : 70 1) 90 (200 to 290) 2) Mean Deviation =∑ (x – x) N Mean – 257, Mean Deviation – 19.8 3) Standard deviation = ∑ ( x – x ) 2 (>30) n Standard Deviation = ∑ ( x – x ) 2 (<30) n-1 in this case (10-1) Take the deviation of each value from Mean (x-x) Square each (x – x )2 Add and squared deviation ∑ ( x – x ) 2 Divide by no. of observation or n-1 if <30 Then take square root
  • 152. Answer No for Question No : 71 Resected tissue from OT Chemical disinfect ion and discharge into drain Waste from laboratory Chemical disinfect ion culture / autoclave / microwave and mutilation shredding Needles and syringes Disinfect ion / shredding Discarded medicines Autoclaving / microwaving / incineration Linen contaminated Incineration autoclaving with Blood / Microwaving Used IV set Incineration and drug disposal in secured land fills Liquid waste from house Incineration / deep burial keeping
  • 153. Question No : 71 Write the correct method of treatment and disposable of the following categories of biomedical work? Resected tissue from OT Incineration / deep burial Waste from laboratory Autoclaving / microwaving culture / incineration Needles and syringes Disinfect ion / shredding Discarded medicines Incineration and drug disposal in secured land fills Linen contaminated Incineration, autoclaving with Blood / Microwaving Used IV set Chemical disinfect ion / Autoclaving / microwaving/ Multilation/ shredding Liquid wask from house Chemical disinfect ion keeping and discharge into drain
  • 154. Question No : 72 This child also has joint hypermobility 1) What is the diagnosis? 2) What is the usual mode of inheritance? 3) What is the defect? 4) How many clinical forms? 5) What cardiac among can occur? 6) What surgical emergencies? 7) Difference with Cutis Laxa
  • 155. Answer for Question No: 72 1) Ehlers Danlos 2) AD 3) Defect of fibrillar collagen – quantitative 4) 10 5) MVP, AR 6) Rupture of great vessels, dissecting aneurysm, stroke, rupture of uterus in pregnancy, echymoses, periodantitis. 7) Cutis Laxa – skin hangs in redundant folds – AR EDS – hyperextensible snaps back into place when stretched - AD Cutis Laxa – Bloodhound appearance, aged appearance Hyperelasticity and hypermobility of joints, hoarse cry, lax vocal cords
  • 156. Question No : 73 A child is brought with a history of accidental ingestion of Iron tablets. 1) Which one of the following would be of benefit? Gastric Lavage Activated charcoal Whole bowel irrigation 2) When would you measure serum Iron? 3) What is the level of serum Iron which indicate significant toxicity? 4) If serum iron level reports were delayed, how would you confirm iron ingestion? 5) What are the 2 indications for giving desferioxime? 6) What system exhibits symptom first? When?
  • 157. Answer for Question No : 73 1) Whole bowel irrigation 2) 4 – 8 hrs after ingestion 3) >500µg/dl 4) X – ray abdomen 5) Level >500µg/dl Moderate to severe symptoms 6) GI, 30 minutes to 6 hr
  • 158. Question No : 74 1) What is the diagnosis? 2) What 2 findings are characteristics? 3) What is the effect of treatment? 4) List 4 drugs useful 5) What physical therapy will help?
  • 159. Answer for Question No : 74 1) Psoriasis 2) A) Plaques with yellowish white scale like mica b) Auspitz sign – pinpoint bleed or removal 3) Koebner phenomenon lesions appear 4) Coal tar, topical steroid, salicylic acid, calcipotriene (Vit.D analog), Methotrexate & cyclosporine and retinoid 5) UV light
  • 160. Question No : 75 Assess the development of this 3 year old
  • 161. Answer for Question No : 75 Motor : Rides tricycle, stands on the foot momentarily Adaptive : Tower of 10 cubes, imitates bridge construction of 3 cubes, copies a circle, imitates a cross Language : Knows age and sex, counts 3 objects correctly, repeats 3 numbers or a sentence of 6 syllables Social : Plays simple games with other children in parallel, helps in dressing – put on shoes, unbuttons, washes hands.
  • 162. Question No : 76 1) What is the diagnosis? 2) Commonest organism? 3) Treatment
  • 163. Answer for Question No : 76 1) Cutaneous larva migrans 2) A. Braziliense (Hook worm of dogs and cats) (other anky + & Strongyloides) 3) Ivermectin – 200mg/kg/one 1 to 2 days Albendazole – 1 OD X 3 days Topical thiabendazole
  • 164. Question No : A mother says she has the following problems in breast- feeding 1) A not enough milk 2) The baby is reluctant to breast feed
  • 165. Answer for Question No : If not enough milk – poor weight ----- <500/--- <125gm/--- < birth weight after 2 weeks <6 times / day urine strong / smelling concentrated urine Common reasons : Poor breast feed p------- : Poor attachment, no night feeds, delayed start, short feeds, rigid schedule, broke, other feeds Psychological – strem, tired Physical causes Baby is illness / con---- among Advice : Refusal or reluctance to breast feed : Baby is in pain, ill, sedation Encourage to ---- and feed more often Use EBM Rooming in Correct positioning Clear ------ nose Treat oral ↓ sedation to mother
  • 166. Question No : 77 Amniocentesis is brief contemplated for pregnant woman for genetic counseling 1) What is the ideal time? 2) What is the most common indication? 3) Name 4 other indications?
  • 167. Answer for Question No : 77 1) 15 to 16 weeks 2) Advanced maternal age > 35 yrs 3) a) Previous child: chromosomal anmaly b) Either parent - a translocation cause c) History of genetic disorder diagnosed by DNA analysis / biochemistry d) Sex detection in XLD / XLR diseases e) Maternal blood testing (triple screening) indication risk f) Work up for fetal anomalies suggested by USG
  • 168. Question No : 79 1) Identify the abnormality in RBC? 2) This child has chronic diarrhea. What is the diagnosis? 3) Which vitamin deficiency in these children is associated with neurological symptom? 4) Which lipid abnormalities are characteristics? 5) What is the fundus finding? 6) What is the mode of inheritance? Answer for Question No : 79
  • 169. 1) Acanthocytosis 2) A Betalipoprotenemia 3) Vit. E 4) Cholesterol TGL Absent B Liproteins 5) Retinitis pigmentosa 6) Autosomal recessive Question No : 78
  • 170. An infant has cough and difficult breathing The respiratory rate 70 / min The infant has severe respiratory distress (head nodding) 1) What does the infant have as per ARI programme? 2) What are other criteria for this status? 3) Will you treat this infant as OP or IP? 4) What is the antibiotic therapy regimen? Answer for Question No : 78 1) Very severe pneumonia
  • 171. 2) Central cyanosis -------- to feed / drunk or vomited everything --------- / lethargy / ----------- 3) IP 4) Ampi + Gent—for 5 days Oral ------- for 5 days Or CM for 10 days Or Ceftriaxone Question No: 80
  • 172. 1) Identify the organism? 2) What 4 stains are used? 3) What is the treatment?
  • 173. Answer for Question No : 80 1) Pneumocystis carinii 2) Grocott- Gomori cyst Toluidine blue Polychrome – Giemsa Trophozoites and sporozoites Fluorescent labeled MAB 3) 5mg/kg once daily 3 day a week Cotrimoxazole 15 – 20mg in 4 3 weeks for AIDS 2 weeks for others Pentamidine
  • 174. Atovaquone, trimextrate + steroids Answer for Question No . 82 1) Giandia Lambia 2) Acute Explosive fowl smelling watering diarrhoea Abd distusion / flatuluce / nansea anorara and epigastic cramps 3) FTT / Lactose Mal Absorbtion / Persistant Steattorrhoea, E hystlylica diarrhoea injection 4) Metronidayole 15 kg